Download as pdf or txt
Download as pdf or txt
You are on page 1of 65

VAJIRAM & RAVI

Institute for IAS Examination

Prelims Test Series - 2024


Full Length Test – 07
(R5537)

Answer Key
&
Detailed Answer Explanation
Prelims Test Series – Full Length Test – 07 (R5537) - Answer Key
1. (d) 11. (c) 21. (a) 31. (c) 41. (d) 51. (b) 61. (b) 71. (a) 81. (c) 91. (c)
2. (c) 12. (b) 22. (a) 32. (b) 42. (c) 52. (c) 62. (c) 72. (a) 82. (d) 92. (a)
3. (d) 13. (b) 23. (b) 33. (b) 43. (c) 53. (c) 63. (c) 73. (d) 83. (d) 93. (b)
4. (a) 14. (c) 24. (a) 34. (b) 44. (c) 54. (b) 64. (b) 74. (a) 84. (d) 94. (b)
5. (b) 15. (a) 25. (b) 35. (c) 45. (b) 55. (a) 65. (a) 75. (c) 85. (b) 95. (d)
6. (b) 16. (b) 26. (d) 36. (b) 46. (d) 56. (d) 66. (a) 76. (d) 86. (c) 96. (b)
7. (d) 17. (a) 27. (d) 37. (d) 47. (d) 57. (c) 67. (a) 77. (c) 87. (a) 97. (b)
8. (c) 18. (c) 28. (d) 38. (b) 48. (c) 58. (b) 68. (a) 78. (b) 88. (a) 98. (c)
9. (b) 19. (b) 29. (a) 39. (b) 49. (b) 59. (d) 69. (b) 79. (b) 89. (b) 99. (b)
10. (c) 20. (a) 30. (d) 40. (c) 50. (c) 60. (a) 70. (b) 80. (b) 90. (a) 100. (a)

Q1.
Answer: d
Explanation:
● The Fundamental Duties in the Constitution of India are inspired by the Constitution of the erstwhile
USSR. They were not present in the original Constitution. The 42nd Constitutional Amendment Act in
1976 added the ten Fundamental Duties of citizens to the Constitution of India. In 2002, one more
Fundamental Duty was added.
● According to Article 51A, it shall be the duty of every citizen of India:
○ to abide by the Constitution and respect its ideals and institutions, the National Flag and the
National Anthem;
○ to cherish and follow the noble ideals that inspired the national struggle for freedom;
○ to uphold and protect the sovereignty, unity and integrity of India;
○ to defend the country and render national service when called upon to do so;
○ to promote harmony and the spirit of common brotherhood amongst all the people of India
transcending religious, linguistic and regional or sectional diversities and to renounce practices
derogatory to the dignity of women;
○ to value and preserve the rich heritage of the country’s composite culture;
○ to protect and improve the natural environment including forests, lakes, rivers and wildlife and
to have compassion for living creatures;
○ to develop scientific temper, humanism and the spirit of inquiry and reform;
○ to safeguard public property and to abjure violence;
○ to strive towards excellence in all spheres of individual and collective activity so that the nation
constantly rises to higher levels of endeavor and achievement; and
○ to provide opportunities for education to his child or ward between the ages of six and fourteen
years. (It was added by the 86th Constitutional Amendment Act, 2002).
Therefore, option (d) is the correct answer.

1
Vajiram & Ravi Prelims Test Series (2024)
Full Length Test – 07 – (R5537)
Q2.
Answer: c
Explanation:
Business Advisory Committee:
● The Business Advisory Committee of Lok Sabha consists of 15 members including the Speaker who is the
ex-officio Chairperson. The members are nominated by the Speaker. In the Rajya Sabha, it has 11
members including the Chairman as its ex-officio chairman. It was constituted for the first time in 1952.
● It regulates the programme and timetable of the House. It recommends the time that should be
allocated for the discussion of the stage or stages of such Government Bills and other business as the
Chairman in consultation with the Leader of the House may direct for being referred to the Committee.
● It may also recommend to the Government to bring forward particular subjects for discussion in the
House and recommend allocation of time for such discussions. The decisions reached by the Committee
are always unanimous in character and representative of the collective view of the House.
Therefore, option (c) is the correct answer.
Relevance: The Rajya Sabha Chairman chaired a meeting of the Business Advisory Committee of Rajya Sabha.

Q3.
Answer: d
Explanation:
● A quasi-judicial power refers to the power vested in the commission established by law, administrative
officers or bodies to determine the rights of those who appear before it. It possesses some powers and
functions of a traditional court. It's not part of the judicial branch of Government, but it can make
decisions that have legally binding effects like courts do.
● Article 280 of the Constitution of India has provided for a Finance Commission as a quasi-judicial body.
It is constituted by the President of India every fifth year or at such earlier time as he considers necessary.
As a quasi-judicial body, it has power in matters related to summoning & enforcing attendance and
requisitioning any public record from any court of office. So, point 1 is correct.
● Article 324 of the Constitution of India has vested in the Election Commission of India (ECI) the
superintendence and control of the entire process for conduct of elections to Parliament and Legislature
of every State and the offices of President and Vice-President of India. Political parties are registered
with the ECI under the law. The commission, as a part of its quasi-judicial jurisdiction, also settles
disputes between the recognised parties. So, point 2 is correct.
● The Central Vigilance Commission (CVC), the main agency for preventing corruption, was established in
1964 by an executive resolution of the Central Government. It is a quasi-judicial, statutory and
regulatory body that plays a crucial role in ensuring transparency and accountability within public
institutions. So, point 3 is correct.
● The Central Information Commission was established by the Central Government in 2005. It was
constituted through an Official Gazette Notification under the provisions of the Right to Information
Act (2005). It is a quasi-judicial authority or tribunal performing judicial functions. It is vested with civil
as well as penal powers. It is required to determine the disputes by balancing the right to privacy and
the right to information. So, point 4 is correct.
So, all four of the above are quasi-judicial bodies in India.
2
Vajiram & Ravi Prelims Test Series (2024)
Full Length Test – 07 – (R5537)
Therefore, option (d) is the correct answer.
Relevance: Punjab and Haryana High Court has asserted that recording reasons is indispensable in the decision-
making process for quasi-judicial authorities while adjudicating on complaints filed before it.

Q4.
Answer: a
Explanation:
Article 355 of the Constitution of India imposes a duty on the Centre to ensure that the Government of every
State is carried on in accordance with the provisions of the Constitution. It is this duty in the performance of
which the Centre takes over the Government of a State under Article 356 in case of failure of Constitutional
machinery in the State. This is popularly known as ‘President’s Rule’. It is also known as a state Emergency’ or
‘Constitutional Emergency’.
● When the President’s Rule is imposed in a State, the President acquires the following extraordinary
powers:
○ S/he can take up the functions of the State Government and powers vested in the Governor or
any other executive authority in the State.
○ S/he can declare that the powers of the State Legislature are to be exercised by the Parliament.
○ S/he can take all other necessary steps including the suspension of the constitutional provisions
relating to any body or authority in the State.
● The State Governor, on behalf of the President, carries on the State Administration with the help of the
Chief Secretary of the State or the advisors appointed by the President. The budget of a State under
President's Rule is presented to the Lok Sabha. The procedure followed with the Budget of the Central
Government is followed in the case of a State Budget also with such variations or modifications, as the
Speaker may make. The Budget passes through several stages of discussion in the Parliament and
particularly in the Lok Sabha.
Therefore, option (a) is the correct answer.
Relevance: The National Commission for Scheduled Castes (NCSC) has recommended President's Rule in West
Bengal amid violence and political tension.

Q5.
Answer: b
Explanation:
Vote on Account:
● According to Article 116 of the Constitution of India, a Vote on Account is an advance grant to cover
short-term expenses until the new financial year begins. The Consolidated Fund of India, as defined in
Article 266, holds all revenue generated by the central government, including taxes and loans. This fund
can't be accessed without proper legal approval, which is obtained each year during the Union Budget.
So, statement 1 is not correct.
● In a year when national elections are happening, the Government doesn't present a full budget but
releases an interim budget instead. A vote-on-account is passed through an interim Budget. It is passed
as a convention without discussion allowing the Government to handle expenses for a short time
leading up to elections. It is because it is unfair to deny the Government the right to design its budget
for the remainder of the year if it changes after elections. So, statement 2 is correct.
3
Vajiram & Ravi Prelims Test Series (2024)
Full Length Test – 07 – (R5537)
● It is essentially a temporary permission from the Parliament for the Government to spend money,
unlike the full budget, which is a detailed financial statement covering expenses, income and changes
in taxes and policies. Funds made available under Vote on Account are not to be utilized for expenditure
on a ‘New Service’. So, statement 3 is correct.
● Regular budgets are presented for the whole financial year. During the general elections, the incumbent
Government will present an interim budget in place of a full budget. A vote-on-account includes only
the Government's expenditure, whereas the interim budget deals with both receipts and
expenditures.
So, only two of the above statements are correct.
Therefore, option (b) is the correct answer.
Knowledge Box

Interim Budget:
● During the election year, instead of the full Union Budget, the government in power
presents an interim Budget. It is a temporary budget that covers a short period of time
until the election results are out. The Interim Budget gives the government spending
rights till the end of the fiscal year which falls on March 31.
● The first part of the interim Budget is the same as an annual Budget (preceding year's
income and expenses). However, only documentation of the proposed basic expenses
till the elections is included in the interim Budget. In this Budget, any major policy
changes that can influence the voters unfairly are not permitted, as per the Election
Commission's regulations.
Relevance: The Union Finance Minister has recently presented the Interim Budget 2024 along with a Vote on
Account.

Q6.
Answer: b
Explanation:
● Direct democracy is a form of self-government in which all collective decisions are taken through the
participation of all adult citizens of the State in the spirit of equality and open deliberations.
● In a direct democracy, therefore, the best decisions can never be arrived at through voting. The
principle of direct democracy is to govern through consensus, which emerges from careful deliberations
of options or alternatives. In the absence of formal representative institutions, people make decisions
themselves through public discussions.
● In other words, the following principles apply in direct democracy:
○ People are sovereign
○ Sovereignty is inalienable and cannot be represented. So, point 1 is correct.
■ The concept of sovereignty underscores its inalienable nature, signifying that a governing
authority cannot relinquish its power to an external entity nor be adequately represented
by a smaller subset.
○ People must express their general will and make decisions directly through a referendum. So,
point 3 is correct.
4
Vajiram & Ravi Prelims Test Series (2024)
Full Length Test – 07 – (R5537)
○ Decisions are to be based on majority rule. So, point 4 is correct.
● Recall is the name given to a mechanism by which voters can end an elected official's period of office
before the next scheduled election for the office. Combining elements of the initiative process and a
normal candidate election, a recall initiative is launched when a recall motion is filed with the relevant
administration. Proponents are then required to gather a specified number of signatures in support of
the recall measure. So, point 2 is correct.
● To sum up, direct democracy is based on the direct, unmediated and continuous participation of citizens
in the tasks of Governance. It removes the distinction between government and the governed and
between State and civil society. In direct democracy, the State and society become one. It is a system
of popular self-government. So, point 5 is not correct.
Therefore, option (b) is the correct answer.
Relevance: Recently, the Chief Justice of India said that there’s no place for referendums and that the opinion
of the people must be understood via proper institutions.

Q7.
Answer: d
Explanation:
● The Constitution of India divides the powers between the Centre and the States in terms of the Union
List, State List and Concurrent List in the Seventh Schedule. The Union List consists of 98 subjects
(originally 97), the State List 59 subjects (originally 66) and the Concurrent List 52 subjects (originally 47).
○ Both the Centre and the States can make laws on the subjects of the concurrent list, but in case
of a conflict, the Central law prevails. The residuary subjects (i.e., which are not mentioned in any
of the three lists) are given to the Centre.
● Regulation and development of oilfields and mineral oil resources; petroleum and petroleum products;
other liquids and substances declared by Parliament by law to be dangerously inflammable. They form
part of the Union List.
● Preservation, protection and improvement of stock and prevention of animal diseases; veterinary
training and practice. Regulation of mines and mineral development is subject to the provisions of List I
(Union List) concerning regulation and development under the control of the Union. They form part of
the State List.
● Preventive detention for reasons connected with the security of a State, the maintenance of public
order, or the maintenance of supplies and services essential to the community; persons subjected to
such detention form part of the Concurrent List.
● Welfare of labour including conditions of work, provident funds, employers’ liability, workmen’s
compensation, invalidity and old age pensions and maternity benefits form part of the Concurrent List.
Therefore, option (d) is the answer.

Q8.
Answer: c
Explanation:

5
Vajiram & Ravi Prelims Test Series (2024)
Full Length Test – 07 – (R5537)
● Article 243 ZE of the Constitution of India mandates the formation of a Metropolitan Planning
Committee (MPC), for preparing a draft development plan for metropolitan areas in India that have a
population of more than 10 lakh. So, statement 1 is correct.
● Every Metropolitan Planning Committee shall, in preparing the draft development plan have regard to:
○ the plans prepared by the Municipalities and the Panchayats in the Metropolitan area;
○ matters of common interest between the Municipalities and the Panchayats, including
coordinated spatial planning of the area, sharing of water and other physical and natural
resources, the integrated development of infrastructure and environmental conservation.
● The 74th Constitutional Amendment Act states that two-thirds of the members of a Metropolitan
Planning committee should be elected by the elected members of the Municipalities and Chairpersons
of the Panchayats in the metropolitan area from amongst themselves. So, statement 2 is correct.
● The State Legislature may make provisions with respect to the following:
○ composition of such committees;
○ manner of election of members of such committees;
○ representation in such committees of the Central Government, State Government and other
organisations. So, statement 3 is correct.
○ functions of such committees in relation to planning and coordination for the metropolitan area;
and
○ manner of election of chairpersons of such committees.
So, all three of the above statements are correct.
Therefore, option (c) is the correct answer.

Q9.
Answer: b
Explanation:
Secretary-General of Lok Sabha:
● S/he is a permanent official of the Secretariat of the Lok Sabha and is selected and appointed by the
Speaker in consultation with the Leader of the House and the Leader of the Opposition from amongst
those who have made their mark by long years of service in the Parliament or State Legislatures or the
Civil Service. Her/His function is to ensure the administration and working of the Lok Sabha from day to
day and the correct observance of parliamentary procedure. So, statement 1 is not correct.
○ S/he is answerable only to the Speaker and his action cannot be discussed either inside or
outside the House. In his capacity as the Secretary-General of the House, he enjoys the privilege
of freedom from arrest, save on a criminal charge.
○ S/he cannot be obstructed in the execution of his duty as it would amount to contempt of the
House. The House treats as a breach of its privilege not only acts directly tending to obstruct the
Secretary-General or other officers in the performance of their duty but also any conduct which
may tend to deter them from doing their duty in future
● For elections to the Offices of the President and the Vice-President, it has been the established practice
that the Secretary-General of the Lok Sabha or the Rajya Sabha is appointed as Returning Officer by
rotation. So, statement 2 is correct.

6
Vajiram & Ravi Prelims Test Series (2024)
Full Length Test – 07 – (R5537)
● S/he is the permanent link between the changing composition of the different houses and Speakers.
S/he keeps all the parliamentary records and oversees the maintenance of the properties of Parliament
and the functioning of the Parliamentary offices. On behalf of the President of India, he summons
members to attend sessions of Parliament and can authenticate bills in the absence of the Speaker. So,
statement 3 is correct.
● The Secretary-General of Lok Sabha is the Secretary to the forums. S/he acts as the ex officio Secretary-
General of the Inter-Parliamentary Group.
Therefore, option (b) is the correct answer.
Relevance: Recently, the Supreme Court issued notice to the Lok Sabha Secretary General over the expulsion of
a Member of the Parliament.

Q10.
Answer: c
Explanation:
Article 30 of the Constitution of India:
● It grants the following rights to minorities, whether religious or linguistic:
○ The protection under Article 30 is confined only to minorities (religious or linguistic) and does
not extend to any section of citizens (as under Article 29). All minorities shall have the right to
establish and administer educational institutions of their choice. So, statement 1 is correct.
○ The compensation amount fixed by the State for the compulsory acquisition of any property of a
minority educational institution shall not restrict or abrogate the right guaranteed to them. This
provision was added by the 44th Amendment Act of 1978 to protect the rights of minorities in
this regard.
○ In granting aid, the State shall not discriminate against any educational institution managed by a
minority.
● The right under Article 30 also includes the right of a minority to impart education to its children in its
own language. Minority educational institutions are of three types:
○ institutions that seek recognition as well as aid from the State;
○ institutions that seek only recognition from the State and not aid; and
○ institutions that neither seek recognition nor aid from the State.
● The institutions of the first and second types are subject to the regulatory power of the State with
regard to syllabus prescription, academic standards, discipline, sanitation, employment of teaching staff
and so on. The institutions of the third type are free to administer their affairs but subject to the
operation of general laws like contract law, labour law, industrial law, tax law, economic regulations and
so on. So, statement 2 is correct.
● In a judgement delivered in the Secretary of Malankara Syrian Catholic College case (2007), the Supreme
Court has said that Article 30 includes the right of minorities to choose its governing body in whom the
founders of the institution have faith and confidence to conduct and manage affairs of the institution.
So, statement 3 is correct.
So, all three of the above statements are correct.
Therefore, option (c) is the correct answer.

7
Vajiram & Ravi Prelims Test Series (2024)
Full Length Test – 07 – (R5537)
Knowledge Box

● Article 29 of the Constitution grants protection to both religious and linguistic


minorities. Despite common assumptions, the Supreme Court clarified that its scope
isn't limited to minorities only. This is because it refers to a "section of citizens,"
encompassing both minorities and the majority.
Relevance: The Supreme Court has recently said that Article 30 of the Constitution of India is not intended to
segregate communities.

Q11.
Answer: c
Explanation:
● Under the Goods and Services Tax (GST), taxes are calculated on actual revenues after deducting the
taxes paid on inputs. In most cases, refunds are not necessary. However, for exports, which are zero-
rated, exporters don't pay taxes on their output. Instead, they are eligible for refunds on the taxes paid
for their inputs. This forms a significant portion of refunds under the GST system. So, statement 1 is
correct.
● Any supply of goods or services or both to a Special Economic Zone developer/unit will be considered
a zero-rated supply. This means these supplies attract zero tax rates under GST. In other words, supplies
into SEZ are exempt from GST and are considered as exports. So, statement 2 is correct.
Therefore, option (c) is the correct answer.
Relevance: Recently, the Life Insurance Corporation (LIC) received a demand notice for short payment of Goods
and Services Tax (GST).

Q12.
Answer: b
Explanation:
● India is the only country which grows all four species of cotton. i.e. G. Arboreum & G. Herbaceum (Asian
cotton), G. barbadense (Egyptian cotton) and G. Hirsutum (American Upland cotton). G. Hirsutum
represents 90% of India's hybrid cotton production
and all the current Bt cotton hybrids are G.
Hirsutuim. So, statement 1 is correct.
● Analysing the data presented in the table, it is
evident that the production and consumption of
cotton did not follow a consistent upward trend
over the last five years. So, statement 2 is not
correct.
● Similarly, the table illustrates fluctuations in
the import of cotton during the same period,
indicating that it did not exhibit a steady
increase.

8
Vajiram & Ravi Prelims Test Series (2024)
Full Length Test – 07 – (R5537)
● On the other hand, the Minimum Support Price (MSP)
for both Medium and Long Staple cotton exhibited a
consistent and positive trajectory over the last five
years, as evident from the provided data. So,
statement 3 is correct.
So, only two of the above statements are correct.
Therefore, option (b) is the correct answer.
Relevance: The Indian government aims to boost cotton production by encouraging farmers to shift cultivation
from disease-prone to disease-free irrigated areas.

Q13.
Answer: b
Explanation:
● Recently, the Securities and Exchange Board of India (SEBI) has mandated registration of index
providers managing “significant indices” based on securities listed in India to foster transparency in
governance and administration of financial benchmarks in the securities market.
● Index providers are those institutions that formulate and manage indices to support investment needs.
They play a crucial role in categorizing and delineating markets, as their indices serve as representations
of specific markets or segments within them. By defining these markets, index providers establish
benchmarks that enable the assessment of performance within those markets or sectors.
● They have the responsibility to set the rules that decide what securities to include in each index, how
the index will be managed and how securities will be added or removed from that index over time.
● The regulatory framework is in accordance with the International Organisation of Securities
Commission (IOSCO) Principles for Financial Benchmarks.
Therefore, option (b) is the correct answer.
Relevance: SEBI approved a regulatory framework for index providers to foster transparency and accountability
in the governance and administration of financial benchmarks in the securities market.

Q14.
Answer: c
Explanation:
● Internationalisation of the rupee is a process that involves increasing the use of the rupee in cross-
border transactions. It involves promoting the rupee for import and export trade and then other current
account transactions, followed by its use in capital account transactions. These are all transactions
between residents in India and non-residents.
● The use of the rupee in cross-border transactions mitigates currency risk for Indian businesses.
Protection from currency volatility not only reduces the cost of doing business, it also enables better
growth of business, improving the chances for Indian businesses to grow globally. So, statement 1 is
correct.
● While reserves help manage exchange rate volatility and project external stability, they impose a cost
on the economy. Internationalisation of the rupee reduces the need for holding foreign exchange

9
Vajiram & Ravi Prelims Test Series (2024)
Full Length Test – 07 – (R5537)
reserves. Reducing dependence on foreign currency will make India less vulnerable to external shocks.
So, statement 2 is correct.
● Using the rupee for international trade and investment transactions can potentially lower transaction
costs for Indian businesses. It eliminates the need for multiple currency conversions, reducing fees
associated with currency exchange. So, statement 3 is correct.
So, all three of the above statements are correct.
Therefore, option (c) is the correct answer.

Q15.
Answer: a
Explanation:
● The Reserve Bank of India (RBI) is the central bank which is charged with regulating the country’s
currency and credit systems and takes steps through monetary policy to ensure financial stability in India.
Statutory Regulatory Authorities (SRAs) in India are mandated by law to be audited by the Comptroller
and Auditor General (CAG) of India.
● Ordinarily, RBI is audited by two auditors appointed by the Government of India. While the
Government of India retains the authority to appoint the Comptroller and Auditor General (CAG) of India
to scrutinise the accounts of the RBI, this provision has never been exercised. Instead, the established
practice involves chartered accountant firms conducting audits of the RBI.
Therefore, option (a) is the correct answer.
Relevance: The Government of India has appointed two auditors for the audit of the Reserve Bank of India.

Q16.
Answer: b
Explanation:
New Development Bank (NDB):
● It is a multilateral development bank established by Brazil, Russia, India, China and South Africa (BRICS)
to mobilise resources for infrastructure and sustainable development projects in emerging markets and
developing countries.
● Membership of NDB is open to members of the United Nations, borrowing and non-borrowing
members. Current member countries of the NDB include the founding members and new members
namely Bangladesh, United Arab Emirates (UAE) and Egypt. The Bank is headquartered in Shanghai,
China with regional offices in South Africa and Brazil. So, statements 1 and 3 are correct.
● The Bank is governed by a Board of Governors made up of the Finance Ministers of the five BRICS
countries and a Board of Directors. Voting power within the Board is based on each country’s shares in
the bank. So, statement 2 is not correct.
● Russia, India, China and South Africa hold the majority of shares, each contributing approximately
18.98% to the total subscribed capital. Bangladesh, Egypt and UAE also hold shares, contributing smaller
percentages to the overall ownership structure. So, statement 4 is not correct.
○ Russia: Shares Subscribed: 100,000
○ India: Shares Subscribed: 100,000
○ China: Shares Subscribed: 100,000
10
Vajiram & Ravi Prelims Test Series (2024)
Full Length Test – 07 – (R5537)
○ South Africa: Shares Subscribed: 100,000
○ Bangladesh: Shares Subscribed: 9,420
○ Egypt: Shares Subscribed: 11,960
○ United Arab Emirates (UAE): Shares Subscribed: 5,560
So, only two of the above statements are correct.
Therefore, option (b) is the correct answer.
Relevance: The New Development Bank will offer a loan to the Mukhyamantri Gram Sadak Yojana in Gujarat.

Q17.
Answer: a
Explanation:
Reverse Book Building Mechanism:
● It is a mechanism provided for capturing the sell orders on an online basis from the shareholders
through respective Book Running Lead Managers (BRLMs) which can be used by companies intending
to delist their shares through the buyback process. In the Reverse Book Building scenario, the
Acquirer/Company offers to buy back shares from the shareholders.
● It is a process used for efficient price discovery. It is a mechanism where, during the period for which
the Reverse Book Building is open, offers are collected from the shareholders at various prices, which
are above or equal to the floor price. The buyback price is determined after the offer closing date.
Therefore, option (a) is the correct answer.
Relevance: The Securities and Exchange Board of India (SEBI) has recently reviewed the Reverse Book Building
Process.

Q18.
Answer: c
Explanation:
Foreign Currency Convertible Bond (FCCB):
● It is a type of convertible bond issued in a currency different from the issuer's domestic currency. In
other words, the money-issuing company raises it in the form of foreign currency.
○ A convertible bond is a mix between a debt and equity instrument. It acts like a bond by making
regular coupon and principal payments, but these bonds also give the bondholder the option to
convert the bond into stock. These kinds of bonds are often listed by large, multinational
companies with offices around the world, seeking to raise money in foreign currencies.
● It is issued by an Indian company expressed in foreign currency and the principal and interest are
payable in foreign currency.
● These bonds are convertible into equity shares over a period of time as specified in the instrument, they
fall within the purview of the Foreign Direct Investment (FDI) policy and are considered part of the FDI
inflows.
● Further, the bonds are required to be issued under the scheme viz., "Issue of Foreign Currency
Convertible Bonds and Ordinary Shares (through Depository Receipt Mechanism) Scheme, 1993” and
subscribed by a non-resident in foreign currency and convertible into ordinary shares of the issuing
company in any manner. The policy for External Commercial Borrowing is also applicable to FCCBs.

11
Vajiram & Ravi Prelims Test Series (2024)
Full Length Test – 07 – (R5537)
● Companies having pending proceedings for contravention of the Foreign Exchange Management Act,
1999 are not eligible for issuing FCCB.
● The FCCBs bought back/repurchased from the holders must be cancelled and should not be re-issued
or re-sold. The buyback will not have any effect on the bondholders not opting for the buyback or on
the non-participating bondholders of companies opting for the buyback.
Therefore, option (c) is the correct answer.
Knowledge Box

● Indian Depository Receipt: It is an instrument denominated in Indian Rupees in the


form of a depository receipt created by a Domestic Depository (custodian of securities
registered with the Securities and Exchange Board of India) against the underlying
equity of the issuing company to enable foreign companies to raise funds from the
Indian securities markets.
● Masala Bond: They are rupee-denominated bonds issued outside India by Indian
entities. They are debt instruments which help to raise money in local currency from
foreign investors. Both the government and private entities can issue these bonds.
● Additional tier-1 bond: They are unsecured bonds with no pre-determined maturity
date. It forms part of a lender's core capital or commonly known as tier-1 capital.
Relevance: Recently, the Supreme Court ruled against a Special Leave Petition asking that the date of purchasing
the Foreign Currency Convertible Bonds (FCCB) should determine the cost of acquisition.

Q19.
Answer: b
Explanation:
● Three Annual Plans (1966- 69): After the Third Plan's setback, marked by the devaluation of the rupee
to stimulate exports amidst the inflationary recession, the Fourth Five-Year Plan was deferred. The
concurrent agricultural crisis and severe food shortages underscored the urgency of focusing on
agriculture during interim Annual Plans. These interim plans witnessed the adoption of a comprehensive
agricultural strategy, entailing the widespread dissemination of high-yielding seed varieties, increased
fertilizer usage, harnessing irrigation capabilities and prioritizing soil conservation measures. So,
statement 1 is not correct.
○ Third Five-Year Plan (1961-1966): It aimed to make India a 'self-reliant' and 'self-generating'
economy. It failed to achieve its target due to unforeseen events - Chinese aggression (1962),
Indo-Pak war (1965) and severe drought 1965-66. Due to conflicts the approach during the later
phase was shifted from development to defence & development.
● The Annual Plans were prepared between 1966 and 1969 due to two successive years of drought,
devaluation of the currency, a general rise in prices and erosion of resources disrupted the planning
process. The fourth five-year plan started in 1969. So, statement 2 is correct.
● Fifth-Five Year Plan (1974-79): The final Draft of the fifth plan was prepared and launched by D.P. Dhar
in the backdrop of an economic crisis arising out of run-away inflation fuelled by a hike in oil prices and
the failure of the Government takeover of the wholesale trade in wheat. It proposed to achieve two

12
Vajiram & Ravi Prelims Test Series (2024)
Full Length Test – 07 – (R5537)
main objectives: 'removal of poverty' (Garibi Hatao) and 'attainment of self-reliance'. So, statement 3
is correct.
Therefore, option (b) is the correct answer.

Q20.
Answer: a
Explanation:
Ways and Mean Advances (WMA):
● It is a temporary liquidity arrangement with the Central Bank provided under the Reserve Bank of India
(RBI) Act, 1934. It enables the Centre and the States to borrow money for up to 90 days from the RBI to
cover their liquidity mismatch between its inflow of revenues and outflow of expenditure. A higher limit
provides the Government flexibility to raise funds from RBI without borrowing them from the market.
● The interest rate on WMA is the RBI’s repo rate. The Governments are allowed to draw amounts over
their WMA limits. The interest on such overdraft is 2 percentage points above the repo rate. Further,
no State can run an overdraft with the RBI for more than a certain period.
Therefore, option (a) is the correct answer.
Relevance: The Reserve Bank of India (RBI) has recently set the limit for Ways and Means Advances (WMA) for
the second half of the Financial Year 2023-24.

Q21.
Answer: a
Explanation:
Biosphere reserves are sites established by countries
and recognized under UNESCO's Man and the
Biosphere (MAB) Programme to promote
sustainable development based on local community
efforts and sound science. The programme of the
Biosphere Reserve was initiated by UNESCO in 1971.
● Nanda Devi Biosphere Reserve is situated in
the high reaches of Chamoli district in
Uttarakhand. Its core area includes the Nanda
Devi National Park and the Valley of Flowers
National Park.
● Dibru-Saikhowa National Park is located in
Assam. The park is bounded by the
Brahmaputra and Lohit Rivers in the North
and Dibru River in the south. It is the largest
salix swamp forest in North-Eastern India.
● The Nokrek Biosphere Reserve is located on
the Tura Range, which forms part of the
Meghalaya Plateau. The entire area is
mountainous and Nokrek is the highest peak
of the Garo hills. It contains major rivers and
13
Vajiram & Ravi Prelims Test Series (2024)
Full Length Test – 07 – (R5537)
streams that form a perennial catchment system. Examples include the Ganol, Dareng and Simsang
rivers, of which the latter is the longest and largest.
● The Similipal Biosphere Reserve is located in the state of Odisha in Mayurbhanj district. It lies within
two biogeographical regions: the Mahanadian east coastal region of the Oriental realm and the
Chhotanagpur biotic province of the Deccan peninsular zone. It has the largest zone of Sal in all of India.
So, the correct sequence of occurrence of these biosphere reserves as one moves from North to South of India
is 2-1-4-3.
Therefore, option (a) is the correct answer.
Relevance: Recently, World Biosphere Reserve Day was celebrated on November 3 every year.

Q22.
Answer: a
Explanation:
Humboldt’s enigma:
● The theory challenges the notion that biodiversity is solely concentrated in tropical areas. Contrary to
the belief that diversity decreases as one moves away from the tropics, this theory highlights exceptions,
particularly in mountains. It asserts that regions like mountains, situated outside the tropics, can
exhibit significant biodiversity.
● The world’s tropical areas receive more energy from the Sun because of the Earth’s angle of inclination.
So, the tropics have greater primary productivity, which then facilitates greater diversity. More
ecological niches become available, creating more complex ecosystems and greater biological diversity.
● In India, the area lying south of the Tropic of Cancer is supposed to be the most diverse in the country.
The Western Ghats plus Sri Lanka biodiversity hotspot lies in this zone. However, the eastern Himalayas
are much more diverse.
Therefore, option (a) is the correct answer.
Relevance: In the field of ecology, Humboldt's enigma has gained more attention in recent years as researchers
aim to unravel the unexpected biodiversity in mountain ecosystems, which challenges conventional beliefs.

Q23.
Answer: b
Explanation:
● Tiger reserves consist of a Critical Tiger Habitat (CTH) as the core or inviolate area and a buffer area.
They are identified on the basis of the Wildlife (Protection) Act of 1972. As per the Act, establishing the
CTH as ‘inviolate’ involves two steps:
○ Identifying the CTH by establishing on the basis of the scientific and objective area that such areas
are required to be kept as inviolate for the purpose of tiger conservation, without affecting the
rights of the Scheduled Tribes or such other forest dwellers, and notified as such by the State
Government in consultation with an expert committee constituted for the purpose.
○ Establishing the identified CTH as inviolate through voluntary relocation on mutually agreed
terms and conditions.
● As per the Scheduled Tribes And Other Traditional Forest Dwellers (Recognition Of Forest Rights) Act,
2006, Critical Wildlife Habitat (CWH) means such areas of National Parks and Sanctuaries where it has

14
Vajiram & Ravi Prelims Test Series (2024)
Full Length Test – 07 – (R5537)
been specifically and clearly established, case by case, on the basis of scientific and objective criteria,
that such areas are required to be kept as inviolate for wildlife conservation as may be determined and
notified by the Central Government after open process of consultation by an Expert Committee. The
State Government shall initiate the process for notification of a CWH by applying on a case-by-case
basis, to the Union Ministry of Environment, Forests and Climate Change. So, statement 1 is not correct.
● The major difference between the two classifications is that while CTH could be diverted for non-forest
purposes, CWH once notified couldn’t be diverted. CWH as envisaged in the FRA does not necessarily
involve the resettlement of forest-dwellers. It can also be an area of co-existence after modification of
forest rights. So, statement 2 is correct.
● Unlike CTH, the notification of CWH can only be done with the consent of the Gram Sabhas and affected
stakeholders. So, statement 3 is correct.
Therefore, option (b) is the correct answer.
Relevance: The National Tiger Conservation Authority (NTCA) has suggested the Karnataka Government to
notify Nugu Wildlife Sanctuary as a Critical Tiger Habitat (CTH) of the Bandipur Tiger Reserve.

Q24.
Answer: a
Explanation:
● Sacred groves are community or relic forests preserved in the name of religion and culture. They
comprise segments of landscape containing vegetation, animals, and other life forms and geographical
features, protected by human societies. Hunting and logging are prohibited in these forests.
● In India, sacred groves are forest fragments of varying sizes, which are communally protected, usually
having a significant religious connotation for the protecting community.
● The Indian sacred groves are often associated with temples, monasteries, shrines, or burial grounds.
They are known by several names such as Pavitraskhetralu (Andhra Pradesh), Kavu (Kerala), Devaravana
or Devarakadu (Karnataka), Sarana/Jaherthan (Jharkhand), Dev van (Himachal Pradesh),
Devrai/Devgudi (Maharashtra), Ki law Lyngdoh/Ki law Kyntang (Meghalaya), Vanis/Kenkris (Rajasthan),
Kovilkadu (Tamil Nadu), Umanglai (Manipur), Than/Madaico (Assam), and
Garamthan/Harithan/Jahera/Sabitrithan/ Santalburithan (West Bengal).
Therefore, option (a) is the correct answer.
Relevance: During Bhubaneswar's State-level Republic Day Parade, a tableau featured the "Sacred Groves"
concept, with live rituals performed by Santhal community members – a first-time inclusion in the celebration.

Q25.
Answer: b
Explanation:
● The science of reconciliation ecology discovers, designs and deploys productive human habitats that also
support wild native species. Reconciliation ecology studies ways to encourage biodiversity in human-
dominated ecosystems. In reconciled ecosystems, humans dominate. However, biodiversity is allowed
and encouraged to persist within the human-influenced landscape.
● Traditional conservationists believe in the idea of "preservation and restoration". Here, preservation
means keeping pristine lands aside to maintain biodiversity, while restoration means returning the

15
Vajiram & Ravi Prelims Test Series (2024)
Full Length Test – 07 – (R5537)
human-impacted ecosystems to their original state. Reconciliation ecologists argue that these
techniques may not succeed as much of the land has been already impacted and changed substantially
by humans. So, points 2 and 5 are not correct.
● Some of the examples of reconciliation ecology are:
○ In tropical countries, reconciliation ecology is evident through agroforestry practices. Here, crops
or fruit trees are intentionally grown under a canopy of shade trees, creating a habitat for tropical
forest species beyond protected areas. So, point 1 is correct.
○ The application of reconciliation ecology is observed in stormwater treatment ponds. These
ponds serve as breeding habitats for amphibians, particularly in areas where human
development has led to the draining of natural wetlands. So, point 3 is correct.
○ Commercial cardamom plantations exemplify reconciliation ecology by incorporating various
tree species. This intentional diversity aims to provide shade and nectar, supporting threatened
pollinators within the ecosystem. So, point 4 is correct.
Therefore, option (b) is the correct answer.

Q26.
Answer: d
Explanation:
● Turning off the Tap: How the world can end plastic pollution and create a circular economy is a
solutions-focused analysis of concrete practices, market shifts and policies that can inform government
thinking and business action. It is released by the United Nations Environment Programme (UNEP)
which presents a roadmap for tackling the global plastic pollution crisis. It proposes a systemic shift
toward a circular plastics economy, aiming to keep plastics out of the environment and in the economy.
● The report calls for a "systems change" focusing on three key shifts:
o Reduce: Significantly reduce the use of unnecessary and problematic plastic, particularly single-
use items.
o Reuse: Design and use systems that promote the reuse of plastic products and packaging.
o Reorient and Diversify: Move away from fossil fuel-based plastics and invest in more sustainable
alternatives like bioplastics and biodegradable materials.
Therefore, option (d) is the correct answer.
Relevance: Recently the report, “Turning off the Tap”: How the World Can End Plastic Pollution and Create a
Circular Economy, was released by the United Nations Environment Programme (UNEP).

Q27.
Answer: d
Explanation:
● Arsenic is introduced into soil and groundwater during the weathering of rocks and minerals followed
by subsequent leaching and runoff. It can also be introduced into soil and groundwater from
anthropogenic sources. So, point 1 is correct.
● It is emitted into the atmosphere by high-temperature processes such as coal-fired power plants,
burning vegetation (wildfire) and volcanism. In water, particularly groundwater, where there are sulfide
mineral deposits and sedimentary deposits deriving from volcanic rocks, the concentrations can be
16
Vajiram & Ravi Prelims Test Series (2024)
Full Length Test – 07 – (R5537)
significantly elevated. Natural low-temperature biomethylation and reduction of arsines also release
arsenic into the atmosphere. So, points 2, 3 and 4 are correct.
● The occurrence of Arsenic in groundwater is mainly in aquifers up to 100 m depth. The deeper aquifers
are free from Arsenic contamination.
So, all four of the above are the sources of Arsenic pollution.
Therefore, option (d) is the correct answer.
Knowledge Box

● The term “biomethylation” describes the formation of both volatile and non-volatile
methylated compounds of metals and metalloids.
● Arsenic has been shown to cause widespread health effects in humans as a
consequence of exposure to drinking water when present in excessive quantities.
Long-term exposure to arsenic in drinking water is causally related to increased risks of
cancer in the skin, lungs, bladder and kidney, as well as other skin changes such as
hyperkeratosis and pigmentation changes.
● In India, the States of West Bengal, Jharkhand, Bihar, Uttar Pradesh, Assam, Manipur
and Chhattisgarh are reported to be most affected by arsenic contamination of
groundwater above the permissible level.
Relevance: The Union Minister of State for Jal Shakti has informed the Parliament that arsenic has been detected
in groundwater in parts 25 States.

Q28.
Answer: d
Explanation:
Taungya System:
● It is a form of agro-silvicultural land management system mainly practised in forest land. This practice
was developed in Burma (present-day Myanmar) to raise forest plantations virtually free of cost. In India
also the practice has retained the name and is being practised in States like Uttar Pradesh, Madhya
Pradesh and Kerala.
● Under this practise the forest land, after clear felling, is leased to a prospective farmer who is given all
the facilities to stay on the farm and cultivate the clear felled area for a year before he is asked to raise
forest plantations in a specific design as approved by the forest department. As a result of the stored
fertility, the farmer harvests bumper crops for the first year, which is an attraction for the taungyadar.
In the second year, s/he carries out forest plantations along the lines as directed by the forest
department and continues to cultivate in between the plantation lines. In most cases, the cultivation is
continued for a period of one or two years after planting trees because thereafter the plants suppress
the agricultural crop. Thereafter, the taungyadar shifts to another adjacent patch of clear-felled forest
land for carrying out similar operations.
● The system of management is so designed as to permit continuity of work. In some of the states, this
practice has now been given up. The main reason is that the taungyadars neglect tree planting and
concentrate mainly on agriculture and later on refuse to leave the area.

17
Vajiram & Ravi Prelims Test Series (2024)
Full Length Test – 07 – (R5537)
Therefore, option (d) is the correct answer.
Knowledge Box

● A kibbutz is a type of settlement which is unique to Israel. It is a collective community


which is traditionally agrarian.
● The Milpa system is a traditional intercropping system of regional vegetables. Present-
day Mayan farmers cultivate this intercropping system through the practice of slash
and burn together with small plots of other vegetable crops such as chiles, corn, beans,
and squash. It entails a rotation of annual crops with a series of managed and enriched
intermediate stages of short-term perennial shrubs and trees, culminating in the re-
establishment of mature closed forests on the once-cultivated parcel.
● Strip plantation is a type of extension forestry aimed at providing shade to travellers;
enhancing the aesthetic values of roads, railway lines and river banks; augmenting the
supply of fuel wood and fodder; and maintaining ecological balance and greenhouse
effect.
Relevance: In recent years, the taungya system has gained increasing attention from large forest companies as
a tool to contribute to the social well-being of the community.

Q29.
Answer: a
Explanation:
● Losoong: It is a New Year festival celebrated in Sikkim in December. So, pair 1 is not correctly matched.
● Sajibu Cheiraoba: It is celebrated in the State of Manipur in March or April. This festival is marked by
traditional rituals, folk dances, and a sense of community bonding. So, pair 2 is correctly matched.
● Bishuva Sankranti: It refers to the New Year celebrations in Odisha in mid-April. The festival involves
various cultural activities, traditional rituals, and the exchange of special dishes among family and
friends. So, pair 3 is not correctly matched.
● Puthandu: It is celebrated in Tamil Nadu and falls in April. The day is marked by traditional rituals, the
preparation of special dishes and the decoration of homes. So, pair 4 is not correctly matched.
So, only one of the above pairs is correctly matched.
Therefore, option (a) is the correct answer.
Knowledge Box

Other New Year Festivals and States:


● Baisakhi - Punjab
● Navreh - Jammu and Kashmir
● Bohag Bihu - Assam
● Pohela Boishakh - West Bengal
● Vishu - Kerala
● Ugadi - Karnataka
● Bestu Varas - Gujarat

18
Vajiram & Ravi Prelims Test Series (2024)
Full Length Test – 07 – (R5537)
● Gudi Padwa - Maharashtra
Relevance: Recently, the Sajibu Cheiraoba was celebrated in Manipur.

Q30.
Answer: d
Explanation:
Bharatnatyam:
● It is the oldest among all dance forms. It derives its name from Bharat Muni and ‘Natyam’ which means
dance in Tamil. It is often referred to as ‘fire dance’ as it is the manifestation of fire in the human body.
Most of the movements in Bharatnatyam resemble that of a dancing flame.
○ The origins of this dance form can be traced back to ‘Sadir’ – the solo dance performance of the
temple dancers or devadasis’ in Tamil Nadu, hence it was also referred to as ‘Dashiattam.
● In a Bharatnatyam recital, the knees are mostly bent and the weight is equally distributed across both
the feet. It is also characterised by the ‘Ekcharya lasyam’ style in which one dancer plays many different
roles.
● In this dance form, equal emphasis is given on both the Tandava and Lasya aspects of dance, with major
emphasis on ‘mudras’. One of the principle mudras is ‘Kataka mukha hasta’ in which three fingers are
joined to symbolise ‘Om’.
Therefore, option (d) is the correct answer.

Q31.
Answer: c
Explanation:
● Punch-marked coins were issued by various Mahajanapadas. The first Indian punch-marked coins called
Puranas, Karshapanas or Pana, were minted in the 6th century BC by the various Janapadas and
Mahajanapadas of the Indo-Gangetic Plain. So, statement 1 is correct.
o These coins had irregular shapes and standard weights and were made up of silver with different
markings. Saurashtra had a humped bull, Dakshin Panchala had a Swastika and Magadha had
generally five symbols. Magadhan punch-marked coins became the most circulated coins in
South Asia.
o Rupyarupa was the silver punch-marked coin during the Mauryan period. Other coins during
this period were suvarnarupa (gold), tamrarupa (copper) and sisarupa (lead).
● Indo-Greeks introduced the fashion of showing the bustor head of the ruler on the coins. The legends
on their Indian coins were mentioned in two languages: Greek on one side and in Kharosthi on the other
side of the coin. These coins are significant because they carry detailed information about the issuing
monarch, the year of issue and sometimes an image of the reigning king. Coins were mainly made of
silver, copper, nickel and lead. So, statement 2 is correct.
● The Satavahana kings mostly used lead as a material for their coins. Silver coins were rare. Next to lead,
they used an alloy of silver and copper called ‘potin’. Many copper coins were also available. So,
statement 3 is correct.
So, all three of the above statements are correct.
19
Vajiram & Ravi Prelims Test Series (2024)
Full Length Test – 07 – (R5537)
Therefore, option (c) is the correct answer.

Q32.
Answer: b
Explanation:
● The Jain sacred literature was initially preserved orally from the time of Mahavira. These literatures
were systematised by the Jain councils from time to time. The first systematisation of the Jain canonical
literature took place in a council at Pataliputra (Patna) by the end of the 4th Century B.C. and again in
two other councils in the early 3rd century B.C. in Mathura and Valabhi. The fourth and last Jain council
took place at Valabhi in 454 or 467 A.D. This council is said to be the source of the Svetambara Jain
scripture.
● The Svetambara canon consists of 45 Agamas: 11 Anga (parts) (originally consisted of 12 Angas, the
12th having been lost) 12 Upanga (sub-parts) 4 Mula-Sutra, 6 Chedasutras, 2 chulika- sutras, 10
Prakirnakas (mixed texts). This makes the canon of the Jain (Svetambara) religion.
Therefore, option (b) is the correct answer.

Q33.
Answer: b
Explanation:
Lepakshi Temple Paintings:
● It is located in the Anantapur district of Andhra
Pradesh. These mural paintings were executed
on Veerabhadra temple walls at Lepakshi in the
16th century. They were made during the
Vijayanagara period. So, statement 1 is
correct.
● They have a colossal depiction of Virabhadra
painted on the ceiling of the mandapa. The
scenes depicted here are from the
Mahabharata, the Ramayana cave and the
Puranas. The coronation of Rama, Arjuna
fighting Kirat and Krishna as vatapatrasayi are charming portraits of this series.
o Sittanavasal Cave (Arivar Koil) Paintings are famous rock-cut cave temples are known for the
paintings based on Jainism. These murals have a close resemblance to Bagh and Ajanta
paintings. The paintings are not only on the walls but also on the ceiling and pillars. So,
statement 2 is not correct.
● The paintings show a complete absence of primary colours, especially blue. They depict a decline in
painting in terms of quality. The forms, figures and details of their costumes are outlined in black colour.
So, statement 3 is correct.
So, only two of the above statements are correct.
Therefore, option (b) is the correct answer.

20
Vajiram & Ravi Prelims Test Series (2024)
Full Length Test – 07 – (R5537)
Relevance: The Archaeological Survey of India (ASI) has recently begun restoration work on a mural in the
Veerabhadra Swamy temple in Lepakshi.

Q34.
Answer: b
Explanation:
● Aparajitapriccha was authored by the Bhuvanadeva and deals mostly with architecture in south India.
It describes nine types of rulers: Mahipati, Raja, Naradhipa, Mahamandaleswar, Mandalika,
Mahasamanta, Samanta etc. So, pair 1 is correctly matched.
● Vikramakadevacharita was composed by Bilhana under Vikramaditya VI of the Chalukyas of Kalyani. It
clarifies why the emperor was instructed and guided by Lord Shiva himself to overthrow and replace his
reigning elder brother. So, pair 2 is correctly matched.
● Malatimadhava (Drama of Malati and Madhava) is the most famous play of Bhavabhuti who lived at
the court of Yasovarman of Kannauj. Bhavabhuti was a scholar of 8th-century India. The theme of the
play is original and deals with the ordinary people of society. The plot centres around the love stories of
Malati, daughter of the minister Bhurivasu and Madhava, son of another minister Devarata and
Makaranda and Madayantika, who were Madhava and Malati’s friends respectively. So, pair 3 is not
correctly matched.
○ Dandin (6th and early 7th centuries) was an Indian Sanskrit writer of romances and expounder
on poetics. Scholars attribute to him the Dashakumaracharita and the Kavyadarsha (Mirror of
Poetry).
So, only two of the above pairs are correctly matched.
Therefore, option (b) is the correct answer.

Q35.
Answer: c
Explanation:
● Amir Khusrau (1253-1325 CE) was the most famous Persian writer of this period. He wrote several
poems. He also wrote some Hindi verses. His famous work, the Tughlaq Nama, deals with the rise of
Ghiyasuddin Tughlaq. He enjoyed the patronage of successive rulers of Delhi from Balban to
Ghiyasuddin Tughluq and passed away in 1325.
● Jalal ud-Din Khalji liked poetry and invited many poets to his court. Khusro was admired in his court and
given the title “Amir”. He was given the job of “Mushaf-dar”. Khusro is called the father of qawwali. He
introduced the ghazal style of a song into India. He also introduced many new ragas such as Ghora and
Sanam.
● He was the disciple of Shaikh Nizamuddin Auliya. He gave a unique form to the Chishti order by
introducing the qaul, a hymn sung at the opening or closing of qawwali. This was followed by Sufi poetry
in Persian, Hindavi or Urdu, and sometimes using words from all of these languages.
● His Khazain-ul-Futuh speaks about Alauddin’s conquests. Alauddin Khalji adopted the title of Sikander-
i-Azam and gave Amir Khusrau the title of Tuti-i-Hind (Parrot of India).
● Amir Khusrau was the first Muslim scholar to use Hindi words and idioms in his poems with Indian
themes. He became the first Persian poet who made an effort to introduce Indian elements in his Persian
21
Vajiram & Ravi Prelims Test Series (2024)
Full Length Test – 07 – (R5537)
writings. He experimented with several poetical forms and created a new style of Persian poetry called
Sabaq-i-Hind or the Indian style.
Therefore, option (c) is the answer.
Knowledge Box

Other works of Amir Khusrau:


● He composed some of the great works like Panch Ganj, Matlaul Anwar (The Rising of
the Lights), Shirin wa Khushav, Lila wa Manjum, the Aina-i-Sikandari (The Mirror of
Alexander), the Hasht-Bihist (The Eight Paradises), Qiran-us-sadain (The Conjunction of
two Auspicious Planets), the Tajul-ul-Futuh, the Nuh-Sipihr (The Nine Spheres), the
Tughlaq Nama, the Miftah-ul-Futuh, the Afzal-ul-Fawaid, the Tarikh-i-Dilhi and the
Khazain-ul-Futuh.

Q36.
Answer: b
Explanation:
● Kalhana’s Rajtarangini distinguishes between Svamandal and mandalantar, which suggests that Kings
in Kashmir exercised direct administrative control mostly over their mandalas while other mandalas
were governed by samantas with an obligation to pay tribute and a commitment of allegiance.
● At the lowest level perhaps villages also may have acquired a lord either by land grants placing villages
under samanta chiefs, by forceful occupation or by the submission of individuals. There is a reference to
qulma as consisting of three to five villages. Allusions to gramapati, gramadhipati, dashagramapati,
vimshatimsha-gramapati, sahasragramapati indicate a hierarchy of villages.
● The mode and quantum of payment to village heads were also specified. Dashasha (head of ten villages)
got as much land as he could cultivate with one plough. Vimshatisha (head of twenty villages) got land
cultivable through four ploughs. Shatesha (head of one hundred villages) was to master one full village
as his remuneration.
Therefore, option (b) is the correct answer.

Q37.
Answer: d
Explanation:
● Uranus is approximately 4 times the size of Earth and 14.536
times as massive. However, as a gas giant, its density is
significantly lower than Earth's. As a result, its surface gravity
(measured from its cloud tops) is slightly weaker than that of
Earth. So, statement 1 is not correct.
● Gravity is a fundamental force of physics. The magnitude of the
force is directly proportional to the masses of the two objects
and inversely proportional to the square of the distance between

22
Vajiram & Ravi Prelims Test Series (2024)
Full Length Test – 07 – (R5537)
the two objects. Depending on the size, mass and density of the object, the gravitational force acting
upon it varies. So, statement 2 is correct.
So, statement-I is incorrect but Statement-II is correct.
Therefore, option (d) is the correct answer.
Knowledge Box

● Newton envisioned gravity as a tug of war along the line connecting any two objects in
the universe. In contrast, Einstein envisioned gravity as a bending of space-time by
mass.
● NASA’s Gravity Probe B (GP-B) mission has confirmed two key predictions derived from
Albert Einstein’s General Theory of Relativity. The probe was launched in 2004. It
carried four ultra-precise gyroscopes designed to measure two effects hypothesized by
Einstein’s theory:
○ The geodetic effect is the warping of space and time by the gravitational field of
a massive body.
○ The frame-dragging effect is the amount by which a spinning object pulls space
and time with it as it rotates.
Relevance: Recently, an astrophysicist has proposed a new theory of gravity without a conservation law.

Q38.
Answer: b
Explanation:
● Red Sanders is native and endemic to India and can only be found in the Southern parts of the Eastern
Ghats. There is a huge demand for this timber in the domestic and international markets, especially in
East Asian Countries. It is used for carvings, furniture, poles and house posts. The rare “wavy” grain
variant is highly valued in Japan for its acoustic properties and is used to make musical instruments. So,
statement 1 is correct.
● Under the foreign trade policy of India, the import of Red Sanders is prohibited, while export is
restricted. Despite the regulation and legal protection, illegal logging and timber extraction remain a
major conservation concern for the species. So, statement 2 is not correct.
● It is listed as endangered as per the International Union for Conservation of Nature (IUCN) Red List of
Threatened Species. It was listed because of over-exploitation in South India. Following the amendment
to the Wildlife (Protection) Act 1972 in 2022, it is now listed in Schedule IV of the Wildlife Protection
Act, 1972. So, statement 3 is not correct.
● It is a very slow-growing tree species that attains maturity in natural forests after 25 – 40 years. Its
heartwood is known to have the capacity to accumulate heavy and rare earth elements such as strontium
cadmium, zinc, and copper. 'Santalin' is a natural dye obtained from its heartwood and is used in
pharmaceutical preparations, food articles, leather and textile industries. So, statement 4 is correct.
So, only two of the above statements are correct.
Therefore, option (b) is the correct answer.

23
Vajiram & Ravi Prelims Test Series (2024)
Full Length Test – 07 – (R5537)
Relevance: Recently, the Andhra Pradesh forest department has submitted an application seeking a
Geographical Indication (GI) Tag for Red Sanders.

Q39.
Answer: b
Explanation:
● Collision-Coalescence theory explains that water
droplets in a cloud collide against each other
caused by the convectional currents in the cloud.
As other large water droplets (that are still
suspended in the cloud) collide with small water
droplets, they coalesce (join) causing them to
become larger.
● The diagram explains how the collision and
coalescence take place in the atmosphere thereby
eventually making the droplets heavier enough to
fall as rain.
● The theory best explains the raindrop formation
in warm clouds suggesting that the small droplets in clouds grow larger by collision and coalescence until
they are sufficiently heavy to fall.
Therefore, option (b) is the correct answer.
Knowledge Box

● Cloud seeding process: It is a weather modification technique that improves a cloud’s


ability to produce rain or snow by introducing tiny ice nuclei into certain types of
subfreezing clouds.
● Sublimation-Aggregation Process: It is related to the phase transition from solid to gas
and aggregation involves the clumping of particles.
● Wegener-Bergeron-Findeisen process: It refers to the rapid growth of ice crystals at
the expense of surrounding cloud droplets, which frequently occur in atmospheric
mixed-phase clouds. The process is a result of the difference in saturation vapour
pressures with respect to liquid and ice, and may in some circumstances lead to abrupt
and complete cloud glaciation at temperatures between −40 °C and 0 °C in the Earth's
atmosphere.

Q40.
Answer: c
Explanation:
● The Brahmaputra is one of the largest rivers in the world and ranks fifth with respect to its average
discharge. The river originates from the Chemayungdung Glacier in the Himalayas. After flowing through
Tibet, it enters India through Arunachal Pradesh and flows through Assam and Bangladesh before it
joins the Bay of Bengal.

24
Vajiram & Ravi Prelims Test Series (2024)
Full Length Test – 07 – (R5537)
● The north bank tributaries have very steep slopes and shallow braided channels for a considerable
distance. They have boulder, pebble and coarse sandy beds and carry a heavy silt charge. Generally, they
have flashy floods. They are Subansiri, Ronganadi, Dikrong, Buroi, Borgong, Jiabharali, Dhansiri (North)
Puthimari, Manas, Beki, Aie and Sankosh. So, points 1, 2, 4 and 5 are correct.
○ The Noadehing, Buridehing, Desang, Dikhow, Bhogdoi, Dhansiri (South), Kopilli, Kulsi, Krishnai,
Dhdhnoi and Jinjiran are the main tributaries on the south bank of the river Brahmaputra. So,
point 3 is not correct.
So, only four of the above are the north-bank tributaries of the Brahmaputra River.
Therefore, option (c) is the correct answer.
Knowledge Box

Other facts about the Brahmaputra River:


● The river slope of Brahmaputra is very steep till it enters India which gets significantly
reduced when it enters Assam valley. Due to this sudden flattening of the river slope,
the river becomes braided in nature. During its course in Assam valley, it is joined by
many tributaries. Joining of these tributaries bringing high sediment load activates
braiding.
● All the tributaries of Brahmaputra are rain-fed and foam up with rain. The
precipitation here is mainly due to the South West monsoon. All its tributaries
experience a number of flood waves as per rainfall in respective catchments. If the
flood of the tributaries coincides with the flood of Brahmaputra, it causes severe
problems and devastation.
Relevance: The Asian Development Bank (ADB) has approved a $200 million loan to enhance flood and riverbank
erosion risk management along the Brahmaputra River in Assam.

Q41.
Answer: d
Explanation:
International Date Line (IDL):
● It is located at about 180° east (or
west). It is halfway around the
world from the Prime Meridian (0°
longitude). The date line runs from
the North Pole to the South Pole
and marks the Western and Eastern
Hemisphere divide. It is not straight
but curves around landmasses and
national borders. For example, it
leans towards the east at the
Bering Strait between Asia and
North America, leaving Cape

25
Vajiram & Ravi Prelims Test Series (2024)
Full Length Test – 07 – (R5537)
Dezhnev in Russia a day ahead of Cape Prince of Wales in Alaska even though they are only a few
kilometres apart.
● It is the boundary where each calendar day starts and is also known as the “Line of Demarcation”
because it separates two calendar dates. When a person crosses the IDL while travelling east, a day gets
subtracted (time is lost) and if one crosses the line travelling west, a day is added (time is gained). So,
statement 1 is not correct and statement 2 is correct.
So, statement–I is incorrect but Statement–II is correct.
Therefore, option (d) is the correct answer.
Knowledge Box

● The International Date Line is de facto, which means that it is not based on
international law.
● The Nautical Date Line follows the 180° meridian from the North to the South Pole.
Unlike the International Date Line, the Nautical Date Line is defined by international
agreements.

Q42.
Answer: c
Explanation:
● Residual mountains are evolved by denudation. Where the general level of the land has been lowered
by the agents of denudation, some very resistant areas may remain and these form residual mountains.
They may also evolve from plateaus which have been dissected by rivers into hills and valleys. For
example, Mount Monadnock in the United States of America, the Highlands of Scandinavia, Scotland
and the Deccan Plateau. So, pair 1 is correctly matched.
● Volcanic mountains are built up from material ejected from the fissures in the Earth’s crust. The
materials include molten lava, volcanic bombs, cinders, ashes, dust and liquid mud. They fall around the
vent in successive layers, building up a characteristic volcanic cone. They are often called mountains of
accumulation. They are common in the Circum Pacific Belt and include volcanic peaks like Mount Fuji in
Japan. So, pair 2 is correctly matched.
● Block Mountains are created when large areas are broken and displaced vertically. The uplifted blocks
are termed as horsts and the lowered blocks are called graben. The Rhine Valley and the Vosges
Mountains in Europe are examples of such mountain systems. In India, Vindhya and Satpura are
examples of Block mountains. So, pair 3 is not correctly matched.
● Fold mountains are created where two or more of Earth's tectonic plates are pushed together. At these
colliding, compressing boundaries, rocks and debris are warped and folded into rocky outcrops, hills,
mountains, and entire mountain ranges. Fold mountains are created through a process called orogeny.
Examples include the Himalayas, Rockies, Andes and Alps. So, pair 4 is correctly matched.
So, only three of the above pairs are correctly matched.
Therefore, option (c) is the correct answer.
Relevance: Mount Fuji is facing ecological challenges and safety concerns due to the increasing number of
visitors.

26
Vajiram & Ravi Prelims Test Series (2024)
Full Length Test – 07 – (R5537)
Q43.
Answer: c
Explanation:
● A Bornhardt is an elevated portion of rock with steep sides. It is at least 100 feet high and is dome-
shaped and bare on top. Exfoliation contributes to the formation of bornhardts, one of the most
dramatic features in landscapes formed by weathering and erosion. Most often they are made of granite
or other hard rock, but can also be formed of limestone or sandstone. The shape of a bornhardt is often
irregular. When they exist in a tropical climate, a little vegetation may be seen growing from its surface.
● They are large rock formation that lies below ground within layers of sediment that may slowly become
exposed by erosion. The ground level may drop over thousands of years as sediment surrounding the
rock is washed away by flooding or blown away by the wind.
○ Exfoliation is a form of mechanical weathering in which curved plates of rock are stripped from
the rock below.
● They are normally surrounded by flat plains. They are often found in the desert where there is little
vegetation to protect the ground from erosion. A bornhardt in a tropical or other location may indicate
that the region was once a desert in the past.
Therefore, option (c) is the correct answer.

Q44.
Answer: c
Explanation:
● The Mediterranean Climate or the Warm Temperate Western Margin Climate is found in relatively few
areas in the world. They are entirely continued to the western portion of continental masses, between
30° and 45° north and south of the Equator. The basic cause of this type of climate is the shifting of
wind belts.
● Trees with small broad leaves are widely spaced and never very tall. The absence of shade is a distinct
feature of Mediterranean lands. Plants are in a continuous struggle against heat, dry air, excessive
evaporation and prolonged droughts. These are open woodlands with evergreen oaks. Citrus fruits such
as oranges, figs, olives and grapes are commonly cultivated here because people have removed the
natural vegetation to cultivate what they want. So, statement 1 is correct and statement 2 is not correct.
○ Coniferous forests consist mostly of conifers, which are trees that grow needles instead of leaves
and cones instead of flowers. In conifers forests, the needle-like leaves reduce the surface area.
Their thick cuticle and sunken stomata also help to reduce water loss.
So, statement-I is correct but Statement-II is incorrect.
Therefore, option (c) is the correct answer.
Relevance: Recently, storm Daniel caused extreme flooding across the Mediterranean region.

Q45.
Answer: b
Explanation:
● Monoclonal antibodies (moAbs or mAbs) are proteins made in laboratories that act like antibodies in
human bodies. Antibodies are parts of the immune system. They seek out the antigens (foreign

27
Vajiram & Ravi Prelims Test Series (2024)
Full Length Test – 07 – (R5537)
materials) and stick to them in order to destroy them. They can effectively bind with a part of the viral
envelope that attaches to the human cells to gain entry into the body. This effectively neutralises the
virus. So, statement 1 is not correct.
● MABs are identical copies of an antibody that targets one specific antigen. Scientists can make
monoclonal antibodies by exposing white blood cells to a particular antigen. So, statement 2 is correct.
● They are used for diagnosis, disease treatment and research. They are used as probes to identify
materials in laboratories or for use in home-testing kits like those for pregnancy or ovulation. They are
also used to identify the type of tissue and blood for use in transplants. So, statement 3 is correct.
● Monoclonal antibodies can be engineered to perform various therapeutic functions. This includes
blocking harmful molecules, activating immune responses and delivering targeted therapies to specific
cells or tissues. This versatility is one of the reasons why monoclonal antibodies have become a crucial
tool in medicine.
So, only two of the above statements are correct.
Therefore, option (b) is the correct answer.
Knowledge Box

Other facts about monoclonal antibodies:


● m102.4 is a “potent, fully human” monoclonal antibody that neutralises Hendra and
Nipah viruses, both outside and inside of living organisms. The antibody has passed
phase-one clinical trials. It means that researchers tested it with a relatively small
number of people to estimate the right dose of treatment that also doesn’t cause side
effects.
● The compassionate use of m102.4 has been approved by the Union Government of
India. Since it hasn’t gone through all the stages of the human trial, it will be the choice
of the patient and their family whether they take it. It has to be used in the early stages
of the disease because it does not work once a person already has brain swelling or
encephalitis. For this reason, it has never been used in India.
Relevance: Recently, India has procured monoclonal antibody doses from Australia to combat the Nipah virus
outbreak in Kerala.

Q46.
Answer: d
Explanation:
● Genetic rescue is the process of increasing population growth with new genetic variation by migrating
individuals into another small population (i.e., gene flow). It is understood as increasing genetic diversity
through translocation — by introducing a tiger from a suitably different landscape to bring about a
genetic mix into the tiger population of other protected areas.
● It is often employed in conservation biology to mitigate the negative effects of inbreeding depression,
which can occur when individuals within a population mate with close relatives, leading to decreased
reproductive success and viability of offspring.
● Genetic rescue as mitigation comes with risks of inducing ‘outbreeding depression’ because such
assisted ‘gene flow’ could also introduce other damaging alleles.
28
Vajiram & Ravi Prelims Test Series (2024)
Full Length Test – 07 – (R5537)
Therefore, option (d) is the correct answer.
Relevance: Genetic rescue is proposed as a method to conserve Ranthambore National Park’s tiger population.

Q47.
Answer: d
Explanation:
Direct-to-Mobile (D2M) technology:
● It is a broadcasting technology capable of transmitting multimedia content to consumers' smartphones
without requiring an active internet connection. Traditionally, it has been employed for issuing
emergency alerts and assisting in disaster management. It can also be utilised to enable mobile users
to watch live television channels on their smartphones without an internet connection. So, points 1
and 2 are correct.
● D2M enables governments to share citizen-centric information and telecom operators to provide
multimedia content without straining network bandwidth. Similar to FM radio, it combines broadband
and broadcast, utilizing mobile phones to capture territorial digital TV signals. This benefits consumers
by reducing internet data consumption and allows those in rural areas with limited internet access to
watch video content. For businesses, it helps offload video traffic, decongest mobile networks, reduce
call drops and improve data speeds. So, point 3 is correct.
Therefore, option (d) is the correct answer.
Knowledge Box

5G Broadcast technology:
● An alternative to D2M is 5G Broadcast technology, currently undergoing testing. This
technology relies on high towers with powerful transmitters to distribute media
content via continuous, linear data streams.
● Mobile devices within the transmitter's coverage area can receive these streams in a
receive-only mode. Compatible with both 5G and 4G, this approach eliminates the need
for a new processing unit, potentially reducing device costs.
● Furthermore, the 5G Broadcast network can be deployed within the existing network
setup, commonly used by a majority of broadcasters.
Relevance: Recently, IIT-Kanpur released a white paper on D2M Broadcasting in collaboration with Prasar Bharti
and the Telecommunications Development Society.

Q48.
Answer: c
Explanation:
● The Access, Watch, Reserve (AWaRe) classification of antibiotics was developed in 2017 by the World
Health Organisation (WHO) expert committee on the selection and use of essential medicines as a tool
to support antibiotic stewardship efforts at local, national and global levels.
● The AWaRe classification is intended as a tool for monitoring antibiotic consumption, defining targets
and monitoring the effects of stewardship policies that aim to optimize antibiotic use and curb
antimicrobial resistance.
29
Vajiram & Ravi Prelims Test Series (2024)
Full Length Test – 07 – (R5537)
● Antibiotics are classified into three groups, Access, Watch and Reserve, taking into account the impact
of different antibiotics and antibiotic classes on antimicrobial resistance, to emphasize the importance
of their appropriate use. It is updated every 2 years.
Therefore, option (c) is the correct answer.
Relevance: Less than half of WHO Africa member nations have adopted the WHO Access, Watch, Reserve
(AWaRe) classification of antibiotics in their national essential medicines lists (EML).

Q49.
Answer: b
Explanation:
Advanced Medium Combat Aircraft (AMCA):
● It is envisaged as a 25-tonne twin-engine stealth aircraft with an internal weapons bay and Diverterless
Supersonic Intake which has been developed in India for the first time. It is intended to have an internal
carriage of 1,500 kg of payload and 5,500 kg of external payload with 6,500 kg of internal fuel. So,
statement 1 is correct.
● India's AMCA project is led by the Aeronautical Development Agency (ADA) under the Defence Research
and Development Organisation (DRDO) and aims to elevate the country into the league of nations
possessing fifth-generation fighter capabilities, a domain currently dominated by the US, Russia and
China. It will be manufactured by the Hindustan Aeronautics Limited. So, statement 2 is correct and
statement 3 is not correct.
So, only two of the above statements are correct.
Therefore, option (b) is the correct answer.
Relevance: The Cabinet Committee on Security (CCS) has cleared the project to design and develop India’s fifth-
generation fighter jet Advanced Medium Combat Aircraft (AMCA).

Q50.
Answer: c
Explanation:
● The Samavesha project was recently launched by the Indian Science, Technology and Engineering
Facilities Map (I-STEM) to enhance research collaboration.
○ I-STEM, an initiative of the Office of Principal Scientific Advisor, Government of India.
● The project aims at connecting researchers with scientific institutes through an online portal. The
researcher or the industry looking to avail advanced scientific equipment can connect with the
institution that has the equipment they are looking for and rent it to conduct their experiments through
the I-STEM portal.
● According to I-STEM, it will help in reducing the capital expenditure of researchers, industry and start-
ups for purchasing advanced equipment. At the national level, this prevents duplication of resources in
research institutions.
● The key vision of the Samavesha project includes:
○ Increased indigenous product innovation
○ Knowledge Exchange Ecosystem
○ Making India a hub for ground-breaking discoveries

30
Vajiram & Ravi Prelims Test Series (2024)
Full Length Test – 07 – (R5537)
Therefore, option (c) is the correct answer.
Relevance: Recently, Samavesha project to connect researchers with labs and equipment.

Q51.
Answer: b
Explanation:
Article 368 in Part XX of the Constitution deals with the powers of Parliament to amend the Constitution and
its procedure. It states that the Parliament may, in the exercise of its constituent power, amend by way of
addition, variation or repeal any provision of the Constitution in accordance with the procedure laid down for
the purpose.
Following are some of the most important Constitutional Amendment Acts passed:
● Thirteenth Amendment Act, 1962 gave the status of a State to Nagaland and made special provisions
for it. However, before giving Nagaland the status of the 16th State of the Indian Union, it was placed
under the control of the Governor of Assam in 1961.
○ The Thirty-Fifth Amendment Act, 1974 terminated the protectorate status of Sikkim and
conferred on it the status of an associate state of the Indian Union. So, pair 1 is not correctly
matched.
● Twenty-Fourth Amendment Act, 1971 affirmed the power of Parliament to amend any part of the
Constitution including Fundamental Rights. It also made it compulsory for the President to give his/her
assent to a Constitutional Amendment Bill. So, pair 2 is correctly matched.
● Forty-Fourth Amendment Act, 1978 was mainly passed to nullify some of the other provisions
introduced by the Forty-Second Amendment Act, 1976. It provided that the Fundamental Rights
guaranteed by Article 20 and Article 21 cannot be suspended during a national emergency. It made the
President declare a national emergency only on the written recommendation of the Cabinet. So, pair 3
is correctly matched.
● The Sixty-Ninth Amendment Act, 1991 accorded a special status to the Union Territory of Delhi by
designating it as the National Capital Territory of Delhi. The amendment also provided for the creation
of a 70-member Legislative Assembly and a 7-member council of the Ministers for Delhi. So, pair 4 is not
correctly matched.
So, only two of the above pairs are correctly matched.
Therefore, option (b) is the correct answer.
Knowledge Box

● The territory of Puducherry comprises the former French establishments in India


known as Puducherry, Karaikal, Mahe and Yanam. The French handed over this
territory to India in 1954. Subsequently, it was administered as an ‘acquired territory’,
till 1962 when it was made a Union Territory by the 14th Constitutional Amendment
Act.
Relevance: Recently, the Women’s Reservation Bill became the 106th Amendment Act of the Constitution of
India.

31
Vajiram & Ravi Prelims Test Series (2024)
Full Length Test – 07 – (R5537)
Q52.
Answer: c
Explanation:
● The appointment, posting and promotion of district judges in a State are made by the Governor of the
State in consultation with the High Court.
● A person to be appointed as district judge should have the following qualifications:
○ He should not already be in the service of the Central or the State Government.
○ He should have been an advocate or a pleader for seven years. So, statement 1 is correct.
○ The High Court should recommend him for appointment.
● The district judge is the highest judicial authority in the district. He possesses original and appellate
jurisdiction in both civil as well as criminal matters. In other words, the district judge is also the session
judge. When he deals with civil cases, he is known as the district judge and when he hears the criminal
cases, he is called the session judge. So, statement 2 is correct.
● The session judge has the power to impose any sentence including life imprisonment and capital
punishment (death sentence). However, a capital punishment passed by him is subject to confirmation
by the High Court, whether there is an appeal or not. So, statement 3 is correct.
So, all three of the above statements are correct.
Therefore, option (c) is the correct answer.

Q53.
Answer: c
Explanation:
United Nations Security Council (UNSC):
● It was established by the United Nations (UN) Charter in 1945 which gives primary responsibility for
maintaining international peace and security to the UNSC. It is one of the six principal organs of the UN.
● According to the United Nations Charter, the Security Council first sends a recommendation to the
General Assembly, which will then appoint the Secretary-General. The Secretary-General is appointed
to a five-year term and may serve up to two terms. So, statement 1 is not correct.
● While other organs of the United Nations make recommendations to member states, only the Security
Council has the power to make decisions that member states are then obligated to implement under
the Charter. So, statement 2 is correct.
● Each year the General Assembly elects five non-permanent members (out of 10 in total) for a two-year
term for UNSC. The 10 non-permanent seats are distributed on a regional basis as follows: five for
African and Asian States; one for Eastern European States; two for the Latin American and Caribbean
States; and two for Western European and other States. So, statement 3 is correct.
● Besides the authorization of the use of force by the Security Council, member states can use force when
exercising their right to self-defence according to Article 51 of the United Nations Charter. Member
states can exercise the right to self-defence only in the event of an “armed attack” and “until the
Security Council has taken measures necessary to maintain international peace and security”. So,
statement 4 is correct.
○ Otherwise, the Charter states that all members of the United Nations shall refrain in their
international relations from the threat or use of force against the territorial integrity or political
32
Vajiram & Ravi Prelims Test Series (2024)
Full Length Test – 07 – (R5537)
independence of any State or in any other manner inconsistent with the purposes of the United
Nations.
So, only three of the above statements are correct.
Therefore, option (c) is the correct answer.
Knowledge Box

Workings of the UN General Assembly (UNGA):


● UNGA is the main policy-making organ of the UN. Comprising all Member States, it
provides a unique forum for multilateral discussion of the full spectrum of international
issues covered by the Charter of the United Nations.
● Each of the 193 Member States of the United Nations has an equal vote.
According to the Charter of the United Nations, the General Assembly may:
● Consider and approve the United Nations budget and establish the financial
assessments of Member States
● Consider and make recommendations on the general principles of cooperation for
maintaining international peace and security, including disarmament
● Discuss any question relating to international peace and security except where a
dispute or situation is currently being discussed by the Security Council and make
recommendations on it.
Relevance: Recently, Iraq filed a complaint against Iran at the United Nations Security Council over aggression.

Q54.
Answer: b
Explanation:
● First MINSK Agreement (2014): It was signed between Russia, Ukraine, the Organisation for Security
and Cooperation in Europe (OSCE) and the pro-Russia leaders from Donetsk and Luhansk. It was signed
in Belarus in 2014 seeking an end to the war. It called for an OSCE-monitored ceasefire, exchange of
prisoners, withdrawal of armed formations; military equipment, fighters and mercenaries from Ukraine
and establishing an OSCE-monitored ‘security zone’ along the border. So, pair 1 is correctly matched.
● New START Treaty (2011): It is an agreement for nuclear arms reduction between the United States of
America (USA) and Russia. It establishes limits on deployed strategic nuclear warheads. It is composed
of three tiers of increasing levels of detail: the Treaty text, the Protocol to the Treaty, and the Technical
Annexes. All three tiers are legally binding. Recently, Russia suspended the New START treaty. So, pair
2 is correctly matched.
● Windsor Framework (2023): It was a deal struck between the United Kingdom and the European Union
(EU) regarding post-Brexit trade rules for Northern Ireland. It seeks to address the disruptions to trade
between Northern Ireland and the rest of the United Kingdom (UK). It was caused by the Northern
Ireland Protocol. It aimed at removing the border between Britain and Northern Ireland running through
the Irish Sea. So, pair 3 is not correctly matched.
● INSTEX (Instrument in Support of Trade Exchanges) (2019): It was a European mechanism designed to
bypass U.S. sanctions on trade with Iran by avoiding the use of the dollar. It served as a clearing house,
enabling Iran to trade oil and import goods or services. The shareholders, including several EU
33
Vajiram & Ravi Prelims Test Series (2024)
Full Length Test – 07 – (R5537)
countries, recently decided to liquidate INSTEX. Its Iranian counterpart, the Special Trade and Finance
Instrument (STFI), mirrored its function. So, pair 4 is not correctly matched.
So, only two of the above pairs are correctly matched.
Therefore, option (b) is the correct answer.
Knowledge Box

Northern Ireland Protocol (2021):


● It allowed EU customs rules to apply in Northern Ireland. This was to avoid a hard
customs border between Northern Ireland and the Republic of Ireland. Specifically,
under the Protocol, Northern Ireland would formally be outside the EU single market,
yet EU rules on the free movement of goods and customs union would continue to
apply.
Relevance: Recently, the INSTEX shareholder countries have decided to liquidate it.

Q55.
Answer: a
Explanation:
● Daraa is the southern Syrian region's central city, located next to the border with Jordan. It is known
for its role in the 2011 uprising against President Bashar al-Assad and came under government control
in 2018 through a ceasefire agreement supported by Russia. Recently southern Syria's Daraa province
experienced an explosion that tragically killed a group of children. So, pair 1 is not correctly matched.
● The province of Sistan & Baluchestan is situated in the southeast of the Islamic Republic of Iran. It is
the largest province of Iran. It has a coastal border with the Oman Sea towards the South. In the East, it
borders Pakistan and Afghanistan. On the North it is bordered by the Southern Khorasan Province and
on the West it borders Kerman and Hormozgan provinces. So, pair 2 is not correctly matched.
● Maluku province consists of the southern portion of the Moluccas island group, in eastern Indonesia.
The province virtually encircles the Banda Sea and is bounded to the north and east by the Ceram Sea,
beyond which lie the Indonesian provinces of North Maluku and West Papua, respectively. The province
of Maluku was struck by an earthquake measuring a magnitude of 6.0. So, pair 3 is correctly matched.
So, only one of the above pairs is correctly matched.
Therefore, option (a) is the correct answer.
Relevance: Recently, Pakistan has attacked the hideouts of armed groups in the Sistan-Baluchestan province of
Iran.

Q56.
Answer: d
Explanation:
Central Industrial Security Force (CISF):
● It is an armed force of the Union established under an Act of Parliament, “Central Industrial Security
Force Act, 1968”. It has 74 other formations including 12 reserve battalions and 08 training institutes.
● It came into existence in 1969 to provide integrated security cover to certain sensitive public sector
undertakings. It also has its own Fire Wing which provides services to multiple establishments. CISF
provides security to the strategic establishment, including the Department of Space, the Department of
34
Vajiram & Ravi Prelims Test Series (2024)
Full Length Test – 07 – (R5537)
Atomic Energy, the Airports, the Delhi Metro, the ports, the historical monuments and the basic areas
of the Indian economy such as petroleum and natural gas, electricity, coal, steel and mining. So, points
1, 2, 3 and 4 are correct.
● The CISF also has a specialized VIP Security vertical providing security to important protectees. The
mandate of CISF was broadened to provide security cover to private corporate establishments as well
after the Mumbai terror attacks of 2008. It is the only Central Armed Police Force with a daily public
interface. So, point 5 is correct.
So, the Central Industrial Security Force (CISF) provides security to all of the above.
Therefore, option (d) is the correct answer.
Relevance: Recently, the Union Home Ministry has approved the deployment of the Central Industrial Security
Force (CISF) in the Parliament complex.

Q57.
Answer: c
Explanation:
Field Marshal Rank in India:
● It is the highest attainable rank in the Indian Army. He is ranked as a five-star General Officer and is
ranked above a General. Once a person becomes a Field Marshal, he shall always act as a Field Marshal
as the rank holder carries this rank throughout his life. There is no retirement and they are always
considered as serving officers until their death which includes the payrolls and all other benefits. Field
Marshals are required to be in full uniform during any official occasion. So, statements 1 and 2 are
correct.
● Field Marshal Kodandera Madappa Cariappa was the first Indian Commander-in-Chief of the Indian
Army and led the Indian forces on the Western Front during the Indo-Pakistan War of 1947. He is among
only two Indian Army officers to hold the highest rank of Field Marshal (the other being Field Marshal
Sam Manekshaw). So, statement 3 is correct.
○ Sam Manekshaw played a crucial role during the India-Pakistan war between 1947 and 1948.
The 1962 Indo-China war provided crucial insights into Sam Manekshaw's leadership. One of the
defining moments in his career was his role in the 1971 Indo-Pak war. As the Chief of the Army
Staff, he masterfully led the Indian Army to victory.
So, all three of the above statements are correct.
Therefore, option (c) is the correct answer.
Knowledge Box

● India celebrates Army Day on January 15 to commemorate the achievements of the


first Commander-in-Chief of the Indian Army, General Kodandera Madappa Cariappa.
Later, he went on to become the Field Marshal.
● The equivalent of the Army's Field Marshal, the highest rank attainable in the Air Force
is the Marshal of the Indian Air Force (MIAF) and Arjan Singh was the only person who
held this rank in the history of IAF. It is mostly awarded in a ceremonial capacity. MIAFs
are ranked immediately above the Chief of Air Staff.

35
Vajiram & Ravi Prelims Test Series (2024)
Full Length Test – 07 – (R5537)
● The President of India is the Supreme Commander of the Indian Army, and its
Professional head is the Chief of Army Staff (COAS), who is a four-star general.

Q58.
Answer: b
Explanation:
Khadi and Village Industries Commission (KVIC):
● It was established under the Khadi and Village Industries Commission Act of 1956 and has statutory
recognition. It is an apex organization under the Union Ministry of Micro, Small, and Medium
Enterprises, about khadi and village industries within India. So, statement 1 is not correct.
● It is entrusted with the task of providing financial assistance to institutions and individuals for the
development and operation of Khadi and village industries and guiding them through the supply of
designs, prototypes and other technical information. So, statement 2 is correct.
● It is also charged with the planning, promotion, organisation and implementation of programmes for
the development of Khadi and other village industries in rural areas in coordination with other agencies
engaged in rural development wherever necessary. The Khadi & Village Industries Boards have been set
up to effectively implement the schemes of KVIC in their respective states.
● The Prime Minister's Employment Generation Programme (PMEGP) was launched to generate
employment opportunities in rural as well as urban areas of the country through the setting up of new
self-employment ventures/projects/micro enterprises. It is a central sector scheme administered by the
Union Ministry of Micro, Small and Medium Enterprises (MSME). At the national level, the Scheme is
being implemented by KVIC. So, statement 3 is correct.
Therefore, option (b) is the correct answer.
Knowledge Box

● The following are the schemes implemented by the Khadi and Village Industries
Commission (KVIC):
○ Prime Minister’s Employment Generation Programme (PMEGP)
○ Market Promotion Development Assistance (MPDA)
○ Interest Subsidy Eligibility Certificate (ISEC)
○ Workshed Scheme for Khadi Artisans
○ Strengthening the infrastructure of existing weak Khadi institutions and
assistance for marketing infrastructure
○ Khadi Reform and Development Programme (KRDP)
○ Scheme of Fund for Regeneration of Traditional Industries (SFURTI)
○ Honey Mission
Relevance: Recently, the Quality Council of India and the Khadi and Village Industries Commission (KVIC) has
signed a pact to enhance the quality of khadi products.

36
Vajiram & Ravi Prelims Test Series (2024)
Full Length Test – 07 – (R5537)
Q59.
Answer: d
Explanation:
● ‘Trade in Services for Development’ publication is the result of a joint effort of the World Bank and the
World Trade Organisation (WTO).
● According to a report, India and China have both doubled their global commercial services export shares
between 2005 and 2022. China's share increased from 3.0% to 5.4%, while India's share rose from 2.0%
to 4.4%.
● The remarkable trade performance of developing economies in the expanded services trade measure is
primarily attributed to four leading services exporters and importers: China, Hong Kong, Singapore, and
India.
● It added that India and the Philippines will need to upskill and reskill their workforces and invest in the
development of their domestic services sectors — particularly in terms of R&D to keep pace with rapidly
changing technology and remain competitive and progress up the value chain.
Therefore, option (d) is the correct answer.
Relevance: According to the report "Trade in Services for Development," in India, South Africa, and Turkey, jobs
directly associated with cross-border services exports represent over 10% of total employment in the services
sector.

Q60.
Answer: a
Explanation:
Prepaid Payment Instruments (PPIs):
● They are instruments that facilitate the purchase of goods and services, the conduct of financial services
and enable remittance facilities against the value stored on such instruments. So, statement 1 is
correct.
● PPIs can be issued by banks and non-banks. Banks can issue PPIs after obtaining approval from the
Reserve Bank of India (RBI). The non-bank PPI issuers are companies incorporated in India and
registered under the Companies Act, 1956 / 2013. They can operate a payment system for issuing PPIs
to individuals/organisations after receiving authorisation from RBI. So, statement 2 is correct.
● The following are the three types of Prepaid Payment Instruments (PPIs):
○ Closed system PPIs: These payment instruments do not allow the withdrawal of cash. These
PPIs help in facilitating the purchase of goods and services from that entity only. Besides, these
instruments cannot be used for payments or settlement for third-party services. Examples: Gift
cards
○ Semi-closed system PPIs: These payment instruments do not allow one to make a cash
withdrawal, irrespective of whether they are issued by banks or non-bank institutions. Examples:
Paytm and GPay
○ Open system PPIs: The most commonly used PPIs are debit and credit cards. It allows you to
make a cash withdrawal from these PPIs. So, statement 3 is not correct.
Therefore, option (a) is the correct answer.

37
Vajiram & Ravi Prelims Test Series (2024)
Full Length Test – 07 – (R5537)
Relevance: RBI has proposed permitting the linking of Prepaid Payment Instruments (PPIs) through third-party
UPI applications, thus allowing PPI wallet holders more flexibility to make UPI payments.

Q61.
Answer: b
Explanation:
● Regional Rural Banks (RRBs) are commercial banks owned by the Indian government operating at the
regional level across various states. RRBs were established in 1975 to provide basic banking and financial
services to rural areas, particularly to small and marginal farmers, agricultural labourers, artisans and
small entrepreneurs. They are regulated by the Reserve Bank of India and supervised by the National
Bank for Agriculture and Rural Development (NABARD).
● It was also aimed at establishing a supplementary channel to the 'Cooperative Credit Structure' and
enlarging the institutional credit system for the rural and agriculture sectors. The Government of India,
the concerned State Government and the bank, which had sponsored the RRB contributed to the share
capital of RRBs in the proportion of 50%, 15% and 35%, respectively. The area of operation of the RRBs
is limited to a few notified districts within a State. There are no RRBs in Sikkim, Delhi and Goa. So,
statement 1 is not correct.
● The central government has recently allocated Rs 6212.03 crore to RRBs as part of the recapitalization
scheme aimed at supporting RRBs to maintain the minimum prescribed Capital to Risk-Weighted Assets
Ratio (CRAR) of 9 per cent. So, statement 2 is correct.
● RRBs can issue Interbank Participation Certificates (IBPCs) to Scheduled Commercial Banks in respect
of their priority sector advances above 75 percent of their outstanding advances. So, statement 3 is
correct.
So, only two of the above statements are correct.
Therefore, option (b) is the correct answer.
Knowledge Box

● The Union Government has recently decided to exempt mergers of Regional Rural
Banks (RRBs) ordered by it from the requirement of clearance from the Competition
Commission of India (CCI). The exemption, aimed at fast-tracking such mergers, would
be granted for five years. Relevant sections of the Competition Act will not apply to
these transactions during this period.
Relevance: Recently, the Union Government has decided to exempt mergers of Regional Rural Banks (RRBs)
ordered by it from the requirement of clearance from the Competition Commission of India.

Q62.
Answer: c
Explanation:
● A Preferential Trade Agreement (PTA) involves two or more partners agreeing to reduce tariffs on the
agreed number of tariff lines. The list of products on which the partners agree to reduce duty is called a
positive list. However, in general, PTAs do not cover substantially all trade.
● In a Free Trade Agreement (FTA), tariffs on items covering substantial bilateral trade are eliminated
between the partner countries. However, each partner country maintains an individual tariff structure
38
Vajiram & Ravi Prelims Test Series (2024)
Full Length Test – 07 – (R5537)
for non-members. The key difference between an FTA and a PTA is that while in a PTA there is a positive
list of products on which duty is to be reduced; in an FTA there is a negative list on which duty is not
reduced or eliminated. Thus, compared to a PTA, FTAs are generally more ambitious in coverage of tariff
lines (products) on which duty is to be reduced. So, statement 1 is correct and statement 2 is not correct.
● In an FTA, tariff reduction is generally undertaken with reference to the base rate i.e. from the applied
Most Favoured Nation (MFN) tariffs. However, the World Trade Organisation (WTO) negotiations are
always based on ‘bound tariff’ rates and not the MFN-applied duties. So, statement 3 is correct.
○ MFN tariffs are tariff rates applied by a country on imports from all trading partners that are
members of the WTO unless the country has a Trade Agreement with that country.
Therefore, option (c) is the correct answer.
Knowledge Box

● Most-Favoured Nation (MFN) tariffs are tariff rates a country applies to imports from
all trading partners that are members of the WTO unless the country has a Preferential
Trade Agreement (PTA), like NAFTA, that stipulates different (lower) duties on imports
from specific countries.
○ Under WTO rules, MFN tariffs are the same for all non-preferential partners
(those not part of a preferential trade agreement).
● Bound tariffs are the maximum tariff rate for a given product that a country has
committed not to exceed. The WTO members have the flexibility to apply tariffs at any
level up to their bound level.
Relevance: Recently, India and Switzerland have reached a consensus for a Free Trade Agreement (FTA).

Q63.
Answer: c
Explanation:
● Dovish monetary policy is characterized by measures undertaken by the Reserve Bank of India (RBI) that
promote lower interest rates to stimulate economic activity. In contrast to a hawkish stance, which
focuses on controlling inflation, doves believe in prioritizing economic growth.
● When the RBI increases the repo rate, it signals a hawkish monetary policy stance.
o The repo rate is the rate at which the central bank lends money to commercial banks, and it plays
a crucial role in influencing overall interest rates in the economy. An increase in the repo rate
raises the cost of borrowing for commercial banks. When banks borrow from the central bank at
a higher interest rate, they will likely pass on this increased cost to consumers and businesses by
raising their lending rates. Higher lending rates discourage borrowing and spending by both
individuals and businesses.
● A reduction in the Bank rate is consistent with a dovish approach, as it makes borrowing cheaper and
encourages economic activity.
● When the Reserve Bank of India (RBI) purchases government securities, it is essentially injecting money
into the financial system. This increases the supply of money available for lending and investment. With

39
Vajiram & Ravi Prelims Test Series (2024)
Full Length Test – 07 – (R5537)
more money available, banks have more funds to lend out. This increased supply of funds leads to lower
interest rates because banks compete to offer loans at lower rates to attract borrowers.
● The introduction of Targeted Long-Term Repo Operations (TLTROs) by the RBI is a dovish monetary policy
tool designed to address specific economic challenges and stimulate borrowing. TLROs are Long-term
repo operations (LTROs) conducted by the RBI to ensure adequate liquidity for a longer period for
specific sectors. It provided banks with a liquidity option for the long term unlike short-term policies like
liquidity adjustment facility (LAF) and marginal standing facility (MSF) that the RBI offers to banks.
Therefore, option (c) is the answer.

Q64.
Answer: b
Explanation:
A Geographical Indication (GI) is a sign used on products that have a specific geographical origin and possess
qualities or a reputation that are due to that origin. In order to function as a GI, a sign must identify a product
as originating in a given place. The following products have recently been granted GI Tags:
● Red Ant Chutney or Similipal Kai Chutney is made with a mix of spices and red weaver ants that are
found all year round in the forests of the Mayurbhanj district in Odisha. It is known for its high
nutritional and healing properties. The red weaver ants are called ‘kai pimpudi’ in the local dialect.
These red ants are found in small and big nests that they create by weaving the leaves. So, pair 1 is not
correctly matched.
● Lanjia Saura Painting art form belongs to the 'Lanjia Saura' or 'Lanjia Savara/Sabara' tribe which is one
of the Particularly Vulnerable Tribal Groups (PVTGs) of Odisha. The paintings were originally in the form
of exterior murals in tribal homes. The pattern was white paintings over a crimson maroon background,
So, pair 2 is correctly matched.
● Adi Kekir is a variety of ginger produced in the East Siang, Siang and Upper Siang districts of Arunachal
Pradesh. It is known for its taste and size. It is one of the three items of Arunachal Pradesh that have
recently received the prestigious Geographical Indication (GI) Tag. The other two items are carpets
handmade by Tibetan settlers and wooden items made by the Wancho community. So, pair 3 is not
correctly matched.
● Ramban Anardana refers to pomegranate seeds collected in the Jammu region. In these areas, fruits
are hand-picked when they are ripe and brown-red. Seeds with the pulp are separated by hand from the
rind and are dried in the sun for about two weeks when the colour of the seeds turns reddish brown. It
is used in making chutney and as a souring agent in various preparations. So, pair 4 is correctly matched.
So, only two of the above pairs are correctly matched.
Therefore, option (b) is the correct answer.
Relevance: Recently, the Geographical Indication (GI) Tag was awarded to Simlipal Kai Chutney.

Q65.
Answer: a
Explanation:
● ‘Qanat system’ is an ancient system of drawing water from aquifers. There are bodies of water
underground known as aquifers, some of which can be found at the tops of valleys or near mountains.

40
Vajiram & Ravi Prelims Test Series (2024)
Full Length Test – 07 – (R5537)
A qanat system taps these aquifers and using underground tunnels, moves the water, using gravity,
over many kilometres.
● The qanats have been used for centuries in arid and semi-arid parts of North Africa, the Middle East and
Asia, where water supplies are limited. It’s known by a variety of names, “foggara” in North Africa,
“falaj” in Oman and “qarez” in parts of Asia.
● Qanats require a sloped terrain, such as a mountain or valley, with a specific angle. If the slope is too
steep, erosion and collapse may occur, while if it's not steep enough, water flow might be insufficient,
leading to potential chemical alterations from mineral interactions in the ground.
Therefore, option (a) is the correct answer.
Relevance: The ancient "qanat system," designed to draw water from aquifers, can be used as a potential
solution in arid regions of Africa to address water scarcity challenges.

Q66.
Answer: a
Explanation:
● The Wild Life (Protection) Licencing (Additional Matters for Consideration) Rules, 2024 provide
guidelines on matters to consider before granting licence to those dealing in captive animals, snake
venom, trophy animals and stuffed animals. Until now, the Licensing rules published in 1983 were being
implemented. So, statement 1 is correct.
● As per the new rules, no such licence shall be granted if it relates to any wild animal specified in Schedule
I to the Wildlife Protection Act, 1972, except with the previous consultation of the Central Government.
The licensing restriction for schedule II species has been removed. So, statement 2 is not correct.
● It mentioned that in additional matters to consider granting licences, the authorised officers must
consider the capacity of the applicant to handle the business concerned in terms of facilities, equipment
and feasibility of premises for the business.
● As per the revised amendments, Schedule I includes animals requiring the utmost protection while
Schedule II has species needing comparatively lower protection. Plant species have been listed under
Schedule II while Schedule IV is dedicated to species to be protected under the Convention on
International Trade in Endangered Species of Wild Fauna and Flora (CITES).
Therefore, option (a) is the correct answer.
Relevance: Recently, the Government of India has revised Wild Life (Protection) Licencing (Additional Matters
for Consideration) Rules, 2024 on the wildlife trade, and excluded certain species from licensing restrictions.

Q67.
Answer: a
Explanation:
● Rotifers are microscopic zooplanktons. They can be found in many marine and freshwater
environments and in moist soil, where they inhabit the thin films of water that are formed around soil
particles. The habitat of rotifers may include still water environments, such as lake bottoms, as well as
flowing water environments, such as rivers or streams.

41
Vajiram & Ravi Prelims Test Series (2024)
Full Length Test – 07 – (R5537)
● They can ingest and break down microplastics. However, this revelation introduces a new concern as
these tiny creatures may be exacerbating the plastic threat by transforming the particles into thousands
of smaller and potentially more hazardous nanoplastics.
● Despite their microscopic size, these creatures are abundant, with up to several thousands of individuals
found in one litre of water in a single location.
● They're known for being tough and capable of surviving drying, freezing, starvation and low oxygen.
They can persist for at least 24,000 years in Siberian permafrost.
Therefore, option (a) is the correct answer.
Relevance: Recently, scientists have discovered that rotifers, a type of zooplankton found in marine and
freshwater environments, can ingest and break down microplastics.

Q68.
Answer: a
Explanation:
● The Wetland City Accreditation (WCA) aims to promote the conservation and wise use of urban and
peri-urban wetlands, as well as sustainable socio-economic benefits for local populations. It was
established during the Conference of parties-12 (COP-12) by the Ramsar Convention held in 2015 and
approved a voluntary Wetland City Accreditation system. So, statement 1 is correct.
● Additionally, the Accreditation seeks to encourage cities that are close to and dependent on wetlands
(primarily Wetlands of International Importance, but also wetlands with other conservation category
status) to develop and strengthen a positive relationship with these valuable ecosystems. For formal
accreditation, a candidate for the WCA should satisfy the standards used to implement each of the six
international criteria mentioned in Operational Guidance for WCA of the Ramsar Convention on
Wetlands.
● This voluntary scheme provides an opportunity for cities that value their natural or human-made
wetlands to gain international recognition and positive branding opportunities for their efforts in
demonstrating strong positive relationships with wetlands.
● Indore, Bhopal and Udaipur are the first three cities to be nominated from India for this scheme. Sirpur
Wetland (Ramsar site in Indore), Yashwant Sagar (Ramsar site closer to Indore), Bhoj Wetland (Ramsar
Site in Bhopal), and several Wetlands (Lakes) in and around Udaipur are lifelines to these cities. Puri has
not received this nomination or recognition. So, statement 2 is not correct.
Therefore, option (a) is the correct answer.
Knowledge Box

● The ongoing Amrit Dharohar initiative of the Ministry of Environment, Forest and
Climate Change (MoEFCC) announced as part of the 2023 Budget also aims to achieve
similar goals by promoting the unique conservation values of Ramsar Sites. In this
context, WCA will not only generate public awareness about the conservation of urban
and peri-urban wetlands but will also help in the implementation of Amrit Dharohar
across the country.
Relevance: Recently, the Union Ministry of Environment, Forest and Climate Change submitted proposals for
Wetland City Accreditation for Indore, Bhopal and Udaipur.
42
Vajiram & Ravi Prelims Test Series (2024)
Full Length Test – 07 – (R5537)
Q69.
Answer: b
Explanation:
● Butterflies and moths are both types of insects known as Lepidoptera. They have scaly wings, six legs
and three body parts like other insects.
● Recently, Dusted Apollo, a rare high-altitude butterfly, has been sighted and photographed for the first
time in Himachal Pradesh. It is an extremely rare butterfly and has never been photographed before in
Himachal Pradesh. It is found from Ladakh to West Nepal and it flies between 3,500 to 4,800 meters in
the inner Himalayas.
● Paintbrush Swift Butterfly is a butterfly species of the Hesperiidae family. The species has never been
photographed in Himachal Pradesh since its discovery in 1878. It is the first time that it was
photographed and documented in Himachal Pradesh.
● Neptis Philyra is a rare species of butterfly, commonly known as the long-streak sailor. It has been
recently discovered for the first time in India in Tale Valley Wildlife Sanctuary. To date, the butterfly
species is known to be found across various regions of East Asia, including eastern Siberia, Korea, Japan
and central and southwest China.
Therefore, option (b) is the correct answer.
Relevance: Recently, the paintbrush swift butterfly has been photographed and documented for the first time
in Himachal Pradesh’s Chamba district.

Q70.
Answer: b
Explanation:
● India has set a target to reduce the carbon intensity of the economy by less than 45% by the end of the
decade, achieve 50 per cent cumulative electric power installed by 2030 from renewables, and achieve
net-zero carbon emissions by 2070. It also aims for 500 GW of renewable energy installed capacity by
2030.
● India’s installed non-fossil fuel capacity is about 42% of the country’s total capacity (as of November
2023). The following is the installed capacity for renewables:
○ Solar Power: 73.31 GW
○ Large Hydro: 46.88 GW
○ Wind power: 44.73 GW
○ Biomass/Co-generation: 10.2 GW
○ Small Hydro Power: 4.98 GW
○ Waste To Energy: 0.58 GW
So, the correct sequence of installed capacity of renewables in the ascending order is 2-3-4-5-1.
Therefore, option (b) is the correct answer.
Relevance: India’s share of renewable energy in the energy mix has increased considerably in recent times.

Q71.
Answer: a
Explanation:

43
Vajiram & Ravi Prelims Test Series (2024)
Full Length Test – 07 – (R5537)
● The National Framework for Climate Services in India (NFCS - India) is set up under the auspices of the
Global Framework for Climate Services (GFCS) of the World Meteorological Organization (WMO). It is
premised on strengthening the production, availability, delivery and application of science-based climate
monitoring and prediction services. It will help to strengthen the collaborative efforts between various
stakeholders for delivering full-value chain climate services in India. So, statement 1 is correct.
● Some of the important functions include:
○ Enabling timely and targeted exchange of relevant information
○ Promotion of improved coordination between service providers and user sectors
○ Creation of sustainable platforms to enable timely and targeted exchange of climate information
leading to significant improvement in the delivery of climate services
○ Strengthening of the observational network on land and the seas. So, statement 2 is correct.
○ Improving the data inflow and eventually, using it to run weather and climate models for deriving
climate predictions.
○ Help identify agricultural production, health trends, population distribution in high-risk areas,
road and infrastructure mapping for the delivery of goods and other socio-economic variables.
● The nodal agency for the formulation and implementation of the national framework in India will be the
Indian Meteorological Department (IMD). It will work in bridging functioning gaps between the various
agencies that require climate services. These include hydrological, power, renewable energy, transport,
dams, irrigation and health agencies. So, statement 3 is not correct.
Therefore, option (a) is the correct answer.
Knowledge Box

● The Global Framework for Climate Services (GFCS) is coordinated by the World
Meteorological Organization (WMO). It has been developed to help all climate services
stakeholders join forces and work together with mutually agreed working
arrangements to collectively pursue the common goal of achieving a climate-smart
society. India is a founding member and active co-sponsor of GFCS.
● The five initial priority areas under GFCS are:
○ Disaster risk reduction
○ Agriculture and food security
○ Water resources
○ Public health
○ Energy
Relevance: The India Meteorological Department (IMD) has launched a National Framework of Climate Services
(NFCS) that seeks to reduce losses from climate hazards.

Q72.
Answer: a
Explanation:
● Development of an egg requires exposure to temperature for a considerable length of time. The ideal
incubation temperature for many birds' eggs is about human body temperature. Almost all birds create
44
Vajiram & Ravi Prelims Test Series (2024)
Full Length Test – 07 – (R5537)
the required temperature by sitting on the eggs and incubating them. They transfer heat via a
temporarily bare area of abdominal skin called the brood patch. A few birds, like penguins, pelicans and
gannets, transfer heat through their webbed feet. So, points 1 and 2 are not correct.
● Killdeer and some other shorebirds soak the feathers of their bellies and use them to wet the eggs
before shading, thus helping to cool the developing embryos by evaporative heat loss. So, point 3 is not
correct.
● The Nicobar megapode is endemic to the Nicobar Islands. Unlike most birds that incubate their eggs
using body warmth, megapodes use external heat sources. They build large communal nests, often
referred to as megapode mounds, made of leaves, twigs and other organic materials. The heat
generated by the decaying vegetation helps incubate the eggs. So, point 4 is correct.
So, only one of the above birds uses the heat generated by the decaying vegetation to help incubate the eggs.
Therefore, option (a) is the correct answer.
Relevance: Recently, many Gentoo Penguins died from Bird Flu infection in Antarctica.

Q73.
Answer: d
Explanation:
● The Ranthambore National Park is located in Rajasthan. The Banas River runs through it on the north
and the Chambal River runs through it on the south. So, pair 1 is correctly matched.
● Panna National Park is situated in Madhya Pradesh. It is also a Tiger Reserve. It is situated in the
Vindhyan Ranges. The Ken River flows through the Reserve from south to north and is home to Gharial
and Mugger. It is one of the least polluted rivers and a tributary of Yamuna. So, pair 2 is correctly
matched.
● Bhitarkanika National Park is located in Odisha. It is India’s second-largest mangrove ecosystem after
the Sunderbans. It is situated on a delta formed by rivers, namely Brahmani, Baitarani, and Dhamra.
Gahirmatha Beach, which forms the boundary of the park in the east, is the largest colony of Olive Ridley
Sea Turtles. So, pair 3 is correctly matched.
● Papikonda National Park is situated in Andhra Pradesh. The Godavari River flows through the park. The
park exists in the Eastern Ghats. A unique dwarf breed of goat known locally as the “kanchu mekha”
originates in this region. So, pair 4 is correctly matched.
So, all four of the above pairs are correctly matched.
Therefore, option (d) is the correct answer.
Relevance: Recently, villagers of Khamri in Madhya Pradesh have protested against the Ken-Betwa project in
Panna over compensation for relocation.

Q74.
Answer: a
Explanation:
● Snowfall in Jammu & Kashmir and Ladakh region is crucial for the local climate, winter crops,
horticulture, availability of water in rivers, tourism and for the local economy. Normally, the region gets
its first snowfall in the first half of December and the majority part of January. However, in recent times,

45
Vajiram & Ravi Prelims Test Series (2024)
Full Length Test – 07 – (R5537)
there has been a lack of snowfall in the region including the popular tourist destinations such as
Gulmarg. This lack of snowfall and rainfall can be attributed to the following factors:
○ Western Disturbances: Winter precipitation in the Himalayan region is caused mainly by it.
During winters, about four to six western disturbance events happen every month on average.
However, from 2023 to 2024 there was one feeble western disturbance event in December that
did not bring any rains and another similar one in January. As a result of this, the overall
precipitation during the winter months in the northern regions has also been declining. So, point
1 is correct.
■ Western disturbances (WDs) are low-pressure weather systems that originate in the
Mediterranean Sea and move eastward across the Middle East, Central Asia, and the
Indian subcontinent. They are responsible for changes in weather conditions over the
Indian subcontinent.
○ Positive Phase of El Nino Southern Oscillations (ENSO): El Niño has affected the global
atmospheric circulation, and it has contributed to the deficit precipitation in the region as well.
So, point 2 is not correct.
■ It originates in the equatorial Pacific Ocean and switches between its positive phase (El
Niño) and negative phase (La Niña).
■ ENSO is a recurring climate pattern involving changes in the temperature of waters in the
central and eastern tropical Pacific Ocean.
○ Gradual rise in temperatures: Temperatures in these regions are rising. The rate of increase of
temperature is seen to be higher in the upper elevation areas than in the plains. It is also
contributing to the decline in snowfall.
● Coriolis force is an apparent force caused by the earth’s rotation. It is responsible for deflecting winds
towards the right in the Northern hemisphere and the left in the southern hemisphere. This is also
known as ‘Ferrel’s Law’. It has not been cited as a reason for the lack of snowfall in Jammu and Kashmir.
So, point 3 is not correct.
Therefore, option (a) is the correct answer.
Relevance: There has been a decline in snowfall in Jammu & Kashmir and Ladakh region in 2024.

Q75.
Answer: c
Explanation:
● Ozone is essential to life and acts as a protective shield from harmful ultraviolet radiation. Absence of
the ozone will result in immediate adverse health effects including skin cancer and cataracts. In the
uppermost atmosphere, ozone is formed by the reaction of molecular oxygen with atomic oxygen.
However, it can be depleted due to pollution and other factors.
● When chlorine and bromine atoms come into contact with ozone in the stratosphere, they destroy
ozone molecules. One chlorine atom can destroy over 100,000 ozone molecules before it is removed
from the stratosphere. Ozone can be destroyed more quickly than it is naturally created. Ozone-
Depleting Substances (ODS) that release chlorine include Chlorofluorocarbons (CFCs),

46
Vajiram & Ravi Prelims Test Series (2024)
Full Length Test – 07 – (R5537)
Hydrochlorofluorocarbons (HCFCs), carbon tetrachloride and methyl chloroform. ODS that release
bromine include halons and methyl bromide. So, statement 1 is correct.
● Some natural processes, such as large volcanic eruptions, can have an indirect effect on ozone levels.
They can produce large amounts of tiny particles called aerosols (different from consumer products also
known as aerosols). These aerosols increase chlorine's effectiveness at destroying ozone. The aerosols
in the stratosphere create a surface on which CFC-based chlorine can destroy ozone. However, the effect
of volcanoes is short-lived. So, statement 2 is correct.
● Absence of the ozone will result in immediate adverse health effects including skin cancer and cataracts.
Long-term adverse effects include a diminishing rate of photosynthesis in plants, a decrease in immune
response in humans and an effect on plankton growth. So, statement 3 is not correct.
Therefore, option (c) is the correct answer.
Knowledge Box

● Ozone depletion is not limited to the area over the South Pole.
● The most common form of UV radiation is sunlight, which produces three main types
of UV rays: UVA, UVB and UVC. UVA rays have the longest wavelengths, followed by
UVB and UVC rays which have the shortest wavelengths. Based on their wavelength
these three types of radiation differ in their biological activity and skin penetration. The
UVB and UVC regions are absorbed by ozone, oxygen, water vapour and carbon
dioxide when sunlight passes through the atmosphere. The UVA radiation does not get
significantly filtered by the atmosphere. The shorter wavelength UVC is the most
destructive type of radiation.
Relevance: Recently, the Union Ministry of Environment, Forest and Climate Change (MoEF&CC) celebrated
World Ozone Day.

Q76.
Answer: d
Explanation:
● Fungi are among the most widely distributed organisms on Earth and are of great environmental and
medical importance. Many fungi are free-living in soil or water, while others form parasitic or symbiotic
relationships with plants or animals.
● Fungi are an important part of soil biodiversity and can help tackle global challenges, including climate
change and hunger.
○ They can transform nutrients in a way that makes them available for plants.
○ Some fungi are decomposers and can break down plant and animal debris.
○ They can also propel nitrogen fixation and phosphorus mobilization. So, point 1 is correct.
○ They play a major part in the carbon cycle through the soil food web.
○ Some species live in mutual symbiotic association with plant roots (i.e., mycorrhizal fungi) and
provide more stable carbon stocks. They can perform soil carbon sequestration. So, point 2 is
correct.

47
Vajiram & Ravi Prelims Test Series (2024)
Full Length Test – 07 – (R5537)
○ Some mushrooms (a type of fungus) are commonly found in the diets of many people around
the world.
○ Some fungi species can help in ecosystem restoration by advancing reforestation in degraded
soils and act as pest control seeing that some species are pathogens of arthropods or nematodes.
So, point 3 is correct.
○ Sustainable materials like Mycelium, which is the root structure of mushrooms, can be used as
an alternative to plastic, synthetic and animal-based products. So, point 4 is correct.
So, all four of the above ecosystem services are provided by the fungi.
Therefore, option (d) is the correct answer.

Q77.
Answer: c
Explanation:
● The Ministry of Environment, Forest & Climate Change (MoEF&CC) has notified new stringent emission
norms for coal-based Thermal Power Plants (TPP) for Sulphur Dioxide (SO2) and other pollutants. To
meet such norms, all coal-based thermal power plants have to install a Flue Gas Desulfurization (FDG)
system to control SO2 emissions as per timelines prescribed by the ministry. So, statement 1 is correct.
● There are various types of Flue Gas Desulphurization systems, predominantly the following 03 types of
FGD systems are adopted worldwide:
○ Dry Sorbent Injection: The dry sorbent scrubbing systems are also known as dry injection or spray
drying systems, where SO2 reacts with limestone in flue gas before dust control systems.
○ Wet Limestone Based: It can be installed for the removal of Sulfur Dioxide (SO2) from flue gas
produced by boilers, furnaces and other processes using combustion fuels like coal or fuel oils.
This process can treat a large range of SO2 concentrations with removal efficiencies of up to
99%. Apart from SO2 gases, this system also helps in the substantial reduction of particulate
matter, thus helping in improving the efficiency of dust reduction from power plants. So,
statement 2 is correct.
○ Sea Water Based: It can achieve a 70-95% reduction in SO2 with alkaline seawater and can be
adopted when SO2 emission norms are relaxed. The initial cost of the seawater-based process is
lower than the wet limestone-gypsum FGD process due to the simple equipment configuration.
● The FDG process is done through the addition of absorbents. Substances such as ammonia or sodium
sulphite are used as absorbents; however, the use of lime or limestone slurry (wet limestone scrubbing)
is also widespread. The uncleaned flue gas is sprayed in a scrubber tower (absorber tower) with a mixture
of water and limestone (scrubbing slurry), whereby most of the sulphur dioxide is bonded by a chemical
reaction. So, statement 3 is correct.
So, all three of the above statements are correct.
Therefore, option (c) is the correct answer.
Relevance: Recently, the Union Minister for Power and New & Renewable Energy has informed Lok Sabha about
the installation of Flue Gas Desulphurisation (FGD) equipment in thermal power plants.

48
Vajiram & Ravi Prelims Test Series (2024)
Full Length Test – 07 – (R5537)
Q78.
Answer: b
Explanation:
● Mahatma Jyotiba Phule was educated at a Marathi school with a three-year break at a mission school
in Poona. He concluded that the only way to liberate and raise an oppressed minority was via education
after being inspired by Thomas Paine's book “The Rights of Man”.
● He supported the movement for widow remarriage in 1860 and in 1863, he established a Home for the
prevention of infanticide.
● He presented his testimonies regarding education to the Hunter Commission (Education Commission)
in 1881.
● The British Government appointed him a member of the Poona Municipality in 1876. He continued as
a member till 1882 and fought for the cause of the downtrodden.
● He established Satya Shodhak Samaj in 1873 to organise the lower castes against the Hindu social order
based on varna and caste system. One of his colleagues started the first newspaper of the Movement,
Din Bandhu, in 1877.
● His writings included Brahmanache Kasab (1869) where he exposed the exploitation of Brahmin priests.
In Gulamgiri (1873) he gave a historical survey of the slavery of lower castes. In 1883, he published a
collection of his speeches under the title Shetkaryancha Asud (The cultivator's whip-cord) where he
analysed how peasants were being exploited in those days. Sarvajanik Satyadharma Pustak (A Book of
True Religion For All) was published in 1891 a year after his death.
Therefore, option (b) is the correct answer.
Knowledge Box

Other facts about Mahatma Jyotiba Phule:


● In 1848, he started the first school for girls in Pune. He started a school at Tatyasaheb
Bhide's house. He opened two more schools for girls in 1851. He was honoured by the
Board of Education for the work he did for girls' education in 1852. Phule established
a school for untouchables and a night school in 1852.
● In 1888, Jyotirao Govindrao Phule was given the title of "Mahatma" by social worker
Vithalrao Krishnaji Vandekar.
Relevance: Recently, the birth anniversary of social reformer Jyotiba Phule was celebrated.

Q79.
Answer: b
Explanation:
● Jatra Bhagat, also known as Jatra Oraon, from the Gumla District organized the Oraon tribals to fight
against the oppression being done by the local zamindars and authorities. This movement is known as
the Tana Bhagat movement of 1920-21. In 1921, the tribals actively participated in the Non-Cooperation
Movement. On their persuasion, 'The Bhagat Agricultural Lands Restoration Act' (1948) was passed in
Bihar, for the land-alienated tribals. So, pair 1 is not correctly matched.

49
Vajiram & Ravi Prelims Test Series (2024)
Full Length Test – 07 – (R5537)
○ Budhu Bhagat participated in the Larka Rebellion of 1832. He struggled not only to free the
Chotanagpur region from the British regime but also united the people and led them into a
guerrilla war to raise voices against British injustice.
● Sidhu and Kanhu Murmu participated in the Santhal Hul of 1855-57. In 1855, the Santhals gathered in
the Bhagnadih Village under the leadership of the brothers - Sidhu and Kanhu Murmu - and declared
themselves free from colonial rule. At the outset, British rule was paralyzed in the area and the native
agents were killed. So, pair 2 is correctly matched.
● The Sepoy Mutiny of 1857 was not confined to the activities of Indian Armymen, it spread to the tribal
hinterland as well. One such example is Narayan Singh, the tribal landlord whose ancestors belonged
to the Gond tribal group residing in Sarangarh. In 1856, he relieved the cultivators by distributing grain
hoarded by a merchant - an act of public benefit for which he was publicly executed by the colonial
authorities in 1857 at Raipur. With the assistance of the landlord of Deori, the British forces arrested
Narayan Singh. So, pair 3 is not correctly matched.
● The Koya Revolt of 1862 protested against 'Muttadars' (zamindars) acting as rent collectors for colonial
rulers. The British denied tribals traditional rights over toddy trees, leading to exploitation by traders
who seized produce and cattle through loans. Thamman-Dora led a tribal attack in 1879. Aligning with
Mahatma Gandhiji's Non-Cooperation and Civil Disobedience Movement in 1922-24, Alluri Seetharama
Raju orchestrated guerrilla warfare against the British for two years. So, pair 4 is correctly matched.
So, only two of the above pairs are correctly matched.
Therefore, option (b) is the correct answer.
Knowledge Box

● Koraput Revolt (1942): The Uprising in the Koraput district of Odisha during the Quit
India Movement in 1942 presents an excellent example of the convergence of existing
fituri (uprising) traditions of tribal rebellions with the Pan-India freedom struggle. The
messianic appeal of Gandhiji and other local Congress leaders like Laxman Naik and Lal
Raja united the folks and strengthened the revolt.
● Laxman Naik, a member of the Bhumia tribe in Odisha, gained recognition as the tribal
leader for Koraput and neighbouring areas, including Malkanagiri and Tentulipada.
Leading the Bhumia people, he actively supported the national freedom movement,
rallying them for development initiatives such as road construction, bridge building,
and school establishment.

Q80.
Answer: b
Explanation:
● The Campbell Commission was set up in 1866 in response to the Na Anka famine of Odisha. It killed
one-third of its population. George Campbell, (then a High Court judge) was its President. It
recommended the development of infrastructure such as ports and navigable irrigation canals to make
the region more accessible. So, point 1 is correct.

50
Vajiram & Ravi Prelims Test Series (2024)
Full Length Test – 07 – (R5537)
● Welby Commission was set up by the British Government to investigate wasteful spending in India.
Established in 1895, its official name was the Royal Commission on the Administration of Expenditure of
India. It is claimed that the Commission improved the economic condition of India by reducing excessive
expenditure. So, point 2 is not correct.
● The Woodhead Commission also known as the Famine Inquiry Commission was appointed in 1944 to
investigate the 1943 Bengal Famine. It is said to be a man-made famine. So, point 3 is correct.
○ The Directorate of Plant Protection Quarantine and Storage (DPPQS) was established in 1946 on
the recommendation of the Woodhead Commission.
● The Babington-Smith Commission was appointed in 1919. It directed its attention to the
reestablishment of stability under the then-existing exchange standard. It was against the backdrop of
an unprecedented rise in the price of silver. So, point 4 is not correct.
Therefore, option (b) is the correct answer.

Q81.
Answer: c
Explanation:
Aftermath of the Revolt of 1857:
● The immediate impact of the Revolt was the British Parliament’s decision to terminate the
‘mismanagement’ of Indian affairs by the English East India Company. The Government of India Act,
passed by Parliament in 1858 transferred all the Company’s powers to the British Crown. It declared
Queen Victoria to be the sovereign of British India and provided for the appointment of a Secretary of
State.
● Nominal subservience to the Mughal emperor was removed totally. Widely publicized stories of the
cruelty and atrocities of the sepoys led to an uproar for punishment and retribution among the British
in England and India. They were regarded as ‘disloyal’ and ignominious subjects, who had to be punished
with the greatest severity.
● The act of suppression and punishment accumulated a heavy debt to the Company. It owed most of it
to the British Crown for deploying troops needed to ‘restore order’ which was supposed to be extracted
from India. Moreover, as the East India Company was wound up, compensation to its shareholders also
became a part of the ‘debt’ to be paid by Indian taxpayers.
● The number of Indian soldiers was drastically reduced even as the number of European soldiers
increased. The concept of divide and rule was adopted with separate units being created based on
caste/community/region. Recruits were to be drawn from the ‘martial’ races of Punjab, Nepal and the
North-Western Frontier who had proved loyal to the British during the Revolt. The people of India were
promised freedom of religion without interference from British officials.
Therefore, option (c) is the correct answer.

Q82.
Answer: d
Explanation:

51
Vajiram & Ravi Prelims Test Series (2024)
Full Length Test – 07 – (R5537)
● The book ‘The Indian State’s Problem’ was written by Mahatma Gandhi. It was published in 1941. It was
a compilation of his writings and speeches delivered on numerous occasions. It takes into account
several events related to Gandhi’s association with the Indian freedom struggle.
● It documents the oppressive measures used by the colonial government to silence the people in Awadh
and to advise Congress about their agenda. The book is also a call for the Princely States to unite. It
provides a detailed account of Gandhi’s thoughts that propelled his political decisions and shaped the
independence movement in India.
Therefore, option (d) is the correct answer.

Q83.
Answer: d
Explanation:
● Belan Valley is located in the northern parts of the Vindhyas and the middle part of the Narmada Valley.
It contains evidence of all three phases of Paleolithic settlement, followed by Mesolithic and Neolithic
settlements. Detailed studies have revealed a sequence of stone age industries from the lower
Palaeolithic to Neolithic to Protohistoric. So, statement 1 is correct.
● Homo sapiens first appeared at the end of the Paleolithic phase (Upper Paleolithic). It coincides with
the last phase of the Ice Age when the climate became comparatively warm and less humid. In India,
hominid evidence was discovered in Hathnora in Madhya Pradesh in 1982. The Hathnora specimen or
the Narmada Man dates from the Middle Pleistocene. There is a dispute related to its evolutionary status
whether it is Homo erectus or Archaic Homo sapien. So, statement 2 is correct.
● One of the key aspects of the Mesolithic Age was the reduction in the size of well-established tool
types. The characteristic tools of this age were microliths (miniature stone tools usually made of crypto-
crystalline silica, chalcedony or chert, both of geometrical and non-geometrical shapes). They were not
only used as tools in themselves but also to make composite tools, spearheads, arrowheads and sickles
after having them on wooden or bone handles. So, statement 3 is correct.
● The Neolithic age was innovated in the production of stone tools, producing implements such as
polished, pecked and ground stone tools. They depended on polished stones other than quartzite for
making tools. The use of celts was especially important for ground and polished handaxes. So, statement
4 is correct.
So, all four of the above statements are correct.
Therefore, option (d) is the correct answer.

Q84.
Answer: d
Explanation:
Indus Valley Civilization sites:
● Manda: It is located on the bank of river Chenab, Jammu and Kashmir. It has a three-fold sequence of
culture with sub-periods in the earliest period of occupation.

52
Vajiram & Ravi Prelims Test Series (2024)
Full Length Test – 07 – (R5537)
● Kot Diji: It is an archaeological site located near an ancient flood channel of the Indus River in Pakistan.
It is located to the east of Mohenjodaro. A layer of burned debris separates structures of the early and
the mature periods, which suggests that the settlement was at some point heavily damaged by fire.
● Chanhudaro: It lies on the left bank of the Indus in
present-day Pakistan. It is a tiny settlement as
compared to Mohenjodaro. It was almost exclusively
devoted to craft production, including bead-making,
shell-cutting, metal-working, seal-making and weight-
making.
● Ganweriwala: It is located in Punjab in present-day
Pakistan near the Indian border. It is almost as large
as Mohenjodaro. It is near a dry bed of the former
Ghaggar or Sarasvati River and has not been
excavated yet. It may have been the fifth major urban
centre.
Therefore, option (d) is the correct answer.
Relevance: A research project by the University of Kerala was recently honoured for finding the largest cemetery
in the Indus Valley Civilization.

Q85.
Answer: b
Explanation:
● Vindhyashakti I was the founder of the Vakataka dynasty. He reigned from circa 250 CE to 270 CE. He
probably ruled from Purika. He is credited with having performed many Vedic sacrifices and thus revived
Brahminical rituals, which were in abeyance during the rule of the later Satavahanas.
● Kadamba dynasty was founded in the 4th century CE (345 CE) by Mayurasharman. They ruled over
northern Karnataka and Konkan. They had their capital at Vaijayanti or Banavasi in the north Kanara
district in Karnataka. They were contemporaries of the Pallavas and claimed to be Brahmanas. They
defeated the Pallavas with the help of forest tribes.
● Western Chalukyas dynasty was founded by Pulakeshin I. He established the independent power of this
dynasty. He reigned from circa. 535−566 CE. The capital of his kingdom was Vatapi (Badami, Karnataka).
He performed many shrauta sacrifices such as ashavmedha.
● Dantidurga was the feudatory of the Chalukya king, Kirtivarman II. He later founded the Rashtrakuta
kingdom by taking control of the northern regions of the Chalukya empire and thus ascended to the
throne in circa 733 CE. However, the Rashtrakutas as a kingdom rose in power from c.753 CE.
So, the correct chronological order of the above events, starting from the earliest time is 1-4-2-3.
Therefore, option (b) is the correct answer.

Q86.
Answer: c
Explanation:

53
Vajiram & Ravi Prelims Test Series (2024)
Full Length Test – 07 – (R5537)
● Damodarapura Copper Plate inscriptions were issued by Kumaragupta I. They refer to him as
Maharajadhiraja. It mentions the names of the administrative divisions and officers. Kumaragupta I
himself appointed the Uparika (Governor) of Pundravardhana Bhukti (or Province), the biggest
administrative division in the empire.
● The Bilsad inscription is the oldest record of his Kumaragupta I. A Sanskrit inscription in Gupta's script
documents Dhruvasarman's construction of a temple gateway and other renovations at a temple
dedicated to Lord Subramanya, also known as Brahmanya and Mahasena.
● Kumaragupta I was an emperor of the Gupta Empire of Ancient India. A son of the Gupta emperor
Chandragupta II and Queen Dhruvadevi, he seems to have maintained control of his inherited territory,
which extended from Gujarat in the west to Bengal region in the east.
● The Damodarapura Copper Plate inscriptions mention the trade guilds that served as banks for
advancing loans and receiving deposits in Kumaragupta’s reign. It also stated that the uparika was
assisted in his administrative duties by a corporate council including prominent members of the town.
Therefore, option (c) is the correct answer.

Q87.
Answer: a
Explanation:
● Moving from the Gujarat coast in India towards
Russia via the Red Sea, the first Sea that a ship
encounters will be the Arabian Sea. After
heading westwards, the ship will reach the Gulf
of Aden.
○ Gulf of Aden is located between Yemen
on the south coast of the Arabian
Peninsula and Somalia and Djibouti in
Africa. In the northwest, it connects with
the Red Sea through the Bab el Mandeb
strait.
○ Moving in the northwest direction, the ship will reach the Red Sea. Going straight, the ship will
reach the Gulf of Suez where it has to cross the Suez Canal. Crossing the Canal would bring the
ship to the Mediterranean Sea.
○ The Gulf of Suez is linked to the Mediterranean Sea by the Suez Canal (north) and is an important
shipping route. It is a northwestern arm of the Red Sea between Africa's west and the Sinai
Peninsula (east) of Egypt.

54
Vajiram & Ravi Prelims Test Series (2024)
Full Length Test – 07 – (R5537)
● Again moving in the Northwestern direction will lead the ship to the Sea of Crete. From there, changing
the direction towards the North will
bring the ship into the waters of the
Aegean Sea which is engulfed
between Greece and Turkey.
● Moving in a straight direction, the
ship will encounter Dardanelles
Strait towards the East. After
crossing the Strait, it will reach the
Sea of Marmara. Sailing in a straight
direction, the ship has to cross the
Bosphorous Strait. Upon crossing
this Strait, the ship will reach the
Black Sea. Moving further East, the
ship will reach Sochi city in Russia
which is situated along the Black Sea.
● So, in total, the ship has to sail through three straits, namely, the Bab el Mandeb Strait, Dardanelles
Strait and Bosphorous Strait.
Therefore, option (a) is the correct answer.
Knowledge Box

● Strait of Hormuz is a narrow channel between the Omani Musandam Peninsula and
Iran. It connects the Persian Gulf to the Gulf of Oman. The Strait is deep and relatively
free of maritime hazards. Its depth is greatest near the Musandam Peninsula and tapers
as you move north toward the Iranian shore.
● The Suez Canal is an artificial waterway between southern Asia and northern Africa that
connects the Red Sea to the Mediterranean Sea via the Isthmus of Suez. The canal
substantially reduces the sailing distance between East Asia and Europe and offers a
more direct route between the Indian Ocean and the North Atlantic Ocean through the
Red Sea and the Mediterranean Sea. The Suez Canal, excavated from 1859 to 1869, is
owned by the Government of Egypt and managed by the Suez Canal Authority.
Relevance: Recently, Indian Navy warships were deployed in the Arabian Sea and the Gulf of Aden to prevent
maritime piracy by Somalian pirates.

Q88.
Answer: a
Explanation:
● Alluvial soils dominate regions in India, including Rajasthan, Punjab, Uttar Pradesh, Bihar, West Bengal,
west Assam and north Gujarat. They are agriculturally significant and formed from parent material
transported by water, ice, gravity and wind.

55
Vajiram & Ravi Prelims Test Series (2024)
Full Length Test – 07 – (R5537)
● The soils developed in the western parts of the Indo-Gangetic plains are, however, markedly different
from those in the eastern region. The semi-arid climate of Punjab and adjoining Uttar Pradesh has led
to the development of saline and alkaline soils wherever the subsoil water is high, in contrast to the
absence of such soils in the more humid climate of West Bengal. There is also a greater accumulation
of lime carbonates in the western region. So, statements 1 and 2 are correct.
So, both Statement–I and Statement–II are correct and Statement–II is the correct explanation for Statement–
I.
Therefore, option (a) is the correct answer.

Q89.
Answer: b
Explanation:
Demographics of India:
● As per the United Nations (UN), people under the age of 25 account for more than 40% of India’s
population. One in five people globally who are under the age of 25 live in India. The median age is 28.
By comparison, the median age is 38 in the United States and 39 in China. So, statement 1 is correct.
● According to the United Nations, Adults aged 65 and older comprise only 7% of India’s population as of
2023, compared with 14% in China and 18% in the U.S.
● As per the latest report of the National Family Health Survey - V, the fertility rate in India is higher than
in China and the U.S., but it has declined rapidly in recent decades. At present, the average Indian
woman is expected to have 2.0 children in her lifetime, a fertility rate that is higher than China’s (1.2) or
the United States (1.6), but much lower than India’s in 1992 (3.4) or 1950 (5.9). Total fertility rates also
vary greatly by State in India, from as high as 3 in Bihar and 2.9 in Meghalaya to as low as 1.3 in Goa
and 1.1 in Sikkim. So, statement 2 is correct.
● According to the UN Population Division, more people migrate out of India each year than into it,
resulting in negative net migration. The UN’s medium variant projections suggest India will continue to
experience net negative migration through at least 2100. But India’s net migration has not always been
negative. As recently as 2016, India gained an estimated 68,000 people due to migration (likely to be a
result of an increase in asylum-seeking Rohingya fleeing Myanmar). So, statement 3 is not correct.
So, only two of the above statements are correct.
Therefore, option (b) is the correct answer.

Q90.
Answer: a
Explanation:
● Kallakkadal is essentially coastal flooding during the pre-monsoon (April-May) season by swell waves
on the southwest coast of India. It is a combination of two Malayalam words, including Kallan and Kadal.
“Kallan means thief and Kadal means sea. So, statement 1 is correct.
● It is caused by waves that are formed by an ocean swell. Ocean swells occur not due to the local winds,
but rather due to distant storms like hurricanes, or even long periods of fierce gale winds. During such
storms, huge energy transfer takes place from the air into the water, leading to the formation of very

56
Vajiram & Ravi Prelims Test Series (2024)
Full Length Test – 07 – (R5537)
high waves. Such waves can travel thousands of kilometres from the storm centre until they strike shore.
So, statement 2 is not correct.
● It came under the spotlight after the 2004 tsunami that killed more than 10,000 people. However,
Kallakkadal is often mistaken to be a tsunami, which is a series of enormous waves created by an
underwater disturbance usually associated with earthquakes occurring below or near the ocean. So,
statement 3 is not correct.
So, only one of the above statements is correct.
Therefore, option (a) is the correct answer.
Relevance: Hundreds of houses have been flooded in several coastal areas of Kerala due to high sea waves
called Kallakkadal.

Q91.
Answer: c
Explanation:
● The displacement of rock upward or downward from its original position along a fracture is termed
faulting. The line along which displacement
of the fractured rock strata takes place is
called the fault line.
● Faulting results in the formation of well-
known relief features such as Rift Valleys
and Block Mountains. A rift valley is
formed by the sinking of rock strata lying between two almost parallel faults. (E.g. valley of the Nile, Rift
valley of Narmada and Tapti).
● When rift valleys have steep parallel walls along the fault, they are called Graben. It is a structural
depression formed due to the presence of two normal faults. The uplifted landmass with steep slopes
on both sides is called Horst. It is a special type of reverse fault. So, statements 1 and 2 are correct.
● Escarpment refers to the very steep slope in a continuous line along a fault. Escarpments in alluvium or
recent sediments may be used to ascertain whether a fault is active or inactive. So, statement 3 is
correct.
So, all three of the above statements are correct.
Therefore, option (c) is the correct answer.

Q92.
Answer: a
Explanation:
● Drake Passage is a deep waterway connecting the Atlantic and Pacific oceans between Cape Horn (the
southernmost point of South America) and the South Shetland Islands. It is situated towards the north
of the Antarctic Peninsula. It defines the zone of climatic transition separating the cool, humid, subpolar
conditions and the frigid, polar regions of Antarctica.

57
Vajiram & Ravi Prelims Test Series (2024)
Full Length Test – 07 – (R5537)
● It is believed to be one of the most difficult sea voyages. The winds over the Drake Passage are
predominantly from the west and are most
intense to the north around Cape Horn. Cyclones
formed in the Pacific sweep west to east across
the southern edge of the passage. The strong
temperature contrast between the Antarctic
polar air and warmer maritime air to the north
helps spur strong fronts and frequent cyclonic
storms. So, statements 1 and 2 are correct.
● Another important reason why the Drake
Passage experiences frequent storms is because
the Southern Ocean is unbroken by land. It
implies that winds coming from other regions
remain unrestricted in the absence of any barrier.
So, both Statement-I and Statement-II are correct and Statement-II is the correct explanation for Statement-
I.
Therefore, option (a) is the correct answer.
Relevance: Recently, an expedition company launched flights to Antarctica to avoid rough drake passage.

Q93.
Answer: b
Explanation:
● Banyak Islands: They are part of Indonesia and are located in the northeastern part of the Indian Ocean.
So, point 1 is correct.
● Mayotte Islands: It is an overseas
department and region of France located in
the Indian Ocean, between Madagascar
and the coast of Mozambique. So, point 2
is correct.
● Cocos (Keeling) Islands: It lies in the
eastern Indian Ocean, northwest of the
Australian city of Perth. It comprises coral
atolls and islands, the archipelago includes
North Keeling Island and the South Keeling
Islands. So, point 3 is correct.
● Tuvalu Islands: It is a group of nine tiny
islands in the South Pacific which won
independence from the United Kingdom in
1978. It is the first country in the world to
build a self-digital replica in the metaverse
to preserve its cultural heritage in the face of rising sea levels. So, point 4 is not correct.

58
Vajiram & Ravi Prelims Test Series (2024)
Full Length Test – 07 – (R5537)
● Palau Islands: It is an island country located in the western Pacific Ocean and not the Indian Ocean. So,
point 5 is not correct.
So, only three of the above islands are part of the Indian Ocean.
Therefore, option (b) is the correct answer.
Relevance: To enhance strategic capabilities, Indian military aircraft recently visited Australia's Cocos Islands,
boosting naval and air force interoperability.

Q94.
Answer: b
Explanation:
● A kilonova is a bright blast of electromagnetic
radiation that takes place when two neutron stars or
a neutron star and a stellar-mass black hole collide
and merge. It appears in the sky as a brief flash of light
that peaks and then fades before completely
disappearing. When these collisions occur, a vast
amount of material is ejected from the neutron stars.
This matter is rich in neutrons. The heaviest elements
of the periodic table are forged in this explosion. These
include gold and platinum and radioactive materials
such as uranium, and iodine.
● If it were to occur around 36 light-years or closer from Earth, the resulting radiation from such an event
could cause an extinction-level event. It would emit lethal particles that would decimate Earth's ozone
layer.
Therefore, option (b) is the correct answer.
Relevance: Recently, researchers have developed a method to model the observable signs of a kilonova
explosion from the merger of two orbiting neutron stars.

Q95.
Answer: d
Explanation:
Organic Electrochemical Transistors (OECTs):
● They are an emerging class of transistors based on organic superconducting materials known for their
ability to modulate electrical current in response to small changes in the voltage applied to their gate
electrode. Like other electronics based on organic semiconductors, these transistors could be promising
for the development of various brain-inspired and wearable technologies. So, statement 1 is correct.
● While OECTs offer significant advantages such as promising amplification and sensing abilities, low
driving voltages and a versatile structure, their widespread use has been hampered by limitations
observed in most conventional designs. These drawbacks include instability and slow redox processes,
which can notably compromise their overall performance. So, statement 2 is correct.

59
Vajiram & Ravi Prelims Test Series (2024)
Full Length Test – 07 – (R5537)
● They can be used to create biosensors, wearable devices and neuromorphic systems. However,
restrictions in the micro and nanopatterning of organic semiconductors, as well as topological
irregularities, often limit their use in monolithically integrated circuits. So, statement 3 is correct.
Therefore, option (d) is the correct answer.
Relevance: Recently, researchers at Northwestern University have outlined a novel approach to producing
mechanically flexible and high-density OECTs.

Q96.
Answer: b
Explanation:
Metagenomics:
● It is the study of the structure and function of entire nucleotide sequences isolated and analysed from
all the organisms (typically microbes) in a bulk sample. Metagenomics is often used to study a specific
community of microorganisms, such as those residing on human skin, in the soil or in a water sample.
So, statement 1 is not correct.
● The main difference between genomics and metagenomics is the nature of the sample. Genomics
explores the complete genetic information of a single organism only, whereas metagenomics explores
a mixture of Deoxyribonucleic Acid (DNA) from multiple organisms and entities, such as viruses, viroids
and free DNA. So, statement 2 is correct.
● It includes cutting-edge, futuristic, state-of-art and high-throughput sequencing analysis. It overcomes
the flaws and limitations of conventional microbiology techniques and the new-age 16S rRNA gene
sequencing.
○ Conventional microbiology techniques are time-consuming, inefficient, and less accurate. They
are prone to contamination, give false results and also, need human efforts to elucidate results.
It makes it practically impossible to identify every microbe present in any system.
○ The new-age 16S rRNA gene sequencing can only determine taxonomic information for high-
abundance microbes. Yet again, a huge concentration of the microbial community and less
abundant sequences remained un-studied.
Therefore, option (b) is the correct answer.
Relevance: Recently, scientists from the Nigerian Centre for Disease Control applied metagenomic sequencing
for pathogen surveillance and detection.

Q97.
Answer: b
Explanation:
● Legionnaires' disease: It is a severe form of pneumonia. It is caused by a bacterium known as legionella.
It is caused as a result of interaction with the bacteria from water or soil. Older adults, smokers and
people with weakened immune systems are particularly susceptible to Legionnaires' disease. It can be
fatal if left untreated. So, pair 1 is not correctly matched.
● Meningococcal disease: It is a rare illness produced by bacteria called Neisseria meningitidis. It can
cause death within a day and can result in significant long-term disabilities in survivors. So, pair 2 is
correctly matched.

60
Vajiram & Ravi Prelims Test Series (2024)
Full Length Test – 07 – (R5537)
● Varicella (chickenpox): It is an acute infectious disease caused by varicella-zoster virus (VZV). It is a DNA
virus that is a member of the herpesvirus group. After the primary infection, VZV stays in the body (in
the sensory nerve ganglia) as a latent infection. Reactivation of latent infection causes herpes zoster
(shingles). So, pair 3 is correctly matched.
● Pompe Disease: It is a glycogen storage disease. It is a rare genetic disorder caused by a deficiency of
the enzyme acid Alpha-Glucosidase (GAA). This enzyme is crucial for breaking down glycogen into
glucose within the lysosomes of cells. So, pair 4 is not correctly matched.
So, only two of the above pairs are correctly matched.
Therefore, option (b) is the correct answer.
Relevance: The United States Food and Drug Administration recently approved a meningococcal vaccine.

Q98.
Answer: c
Explanation:
● Quantum Energy Teleportation (QET) is a technique that allows a receiver at a distance to extract the
local energy by Local Operations and Classical Communication (LOCC), using the energy injected by the
supplier as collateral. The key idea behind Quantum Energy Teleportation (QET) is that the energy of any
quantum system is constantly fluctuating. It is these natural energy fluctuations that can be made use
of on a quantum level. So, statement 1 is correct.
○ Teleportation is the ability to send quantum information from one part of the universe to
another, without travelling through the space in between. By sending all the information that
describes a single particle and passing it to another, this second particle takes on all the
characteristics of the first. It is physically indistinguishable from the first and in a sense, becomes
the first particle, albeit in a different part of the universe.
● It exploits quantum entanglement to send encrypted information. Entangled particles remain linked no
matter how far apart they are and a change to one particle always affects its partner in a particular
way. In experiments, for example, a pair of entangled particles is separated and each partner is sent to
a different location. When someone measures the particle at point A, its quantum state is decided and
that event immediately causes a corresponding change in the particle at point B. No physical matter is
transmitted, however, the person at point B can recreate the photon at point A using only information
about the observed changes. So, statement 2 is not correct and statement 3 is correct.
Therefore, option (c) is the correct answer.
Relevance: Recently, scientists have teleported energy over microscopic distances.

Q99.
Answer: b
Explanation:
Haemoglobin (Hb):
● It is an iron-containing protein found in the Red Blood Cells (RBCs) that carries oxygen from the lungs
to the body's tissues and returns carbon dioxide from the tissues back to the lungs. It is made up of four
protein molecules (globulin chains) that are connected together. Haemoglobin forms an unstable

61
Vajiram & Ravi Prelims Test Series (2024)
Full Length Test – 07 – (R5537)
reversible bond with oxygen. In the oxygenated state, it is called oxyhemoglobin and is bright red and
in the reduced state, it is purplish blue. So, statement 1 is correct.
● It helps chondrocytes to cope with low oxygen levels by transporting oxygen within the cells. In the
absence of haemoglobin, chondrocytes experience hypoxic stress, leading to compromised functionality.
Haemoglobin serves as a crucial oxygen reservoir for chondrocytes, ensuring the timely release of oxygen
as required. Without this vital component, chondrocytes struggle to sustain sufficient oxygen levels,
resulting in cellular distress and eventual demise. So, statement 2 is correct.
○ It also plays an important role in maintaining the shape of the RBCs. In their natural shape, RBCs
are round with narrow centres resembling a doughnut without a hole in the middle. Abnormal
haemoglobin structure can, therefore, disrupt the shape of RBCs and impede their function and
flow through blood vessels.
● Haemoglobin levels vary from person to person. Men usually have higher levels than women. A low
haemoglobin level is referred to as anaemia or low red blood count. Women have mean levels
approximately 12% lower than men. So, statement 3 is not correct.
So, only two of the above statements are correct.
Therefore, option (b) is the correct answer.
Knowledge Box

● A White Blood Cell (leukocyte or white corpuscle) is a cellular component of the blood
that lacks haemoglobin. It has a nucleus, is capable of motility, and defends the body
against infection and disease.
● Some of the common causes of anaemia are loss of blood (traumatic injury, surgery,
bleeding, colon cancer, or stomach ulcer), nutritional deficiency (iron, vitamin B12,
folate), bone marrow problems (replacement of bone marrow by cancer), suppression
by red blood cell synthesis by chemotherapy drugs, kidney failure, and abnormal
haemoglobin structure (sickle cell anaemia or thalassemia).
Relevance: A recent study has demonstrated that cartilage-making cells called chondrocytes manufacture and
use haemoglobin.

Q100.
Answer: a
Explanation:
Grand Slam:
● In tennis, the term Grand Slam refers to the accomplishment of winning all four major championships
(Australia, France, Wimbledon and the United States)- in the same calendar season. So, statement 1 is
correct.
○ In running order, the four grand slam events held each year are the: Australian Open, French
Open, Wimbledon and US Open. The French Open is played on clay, Wimbledon on grass and the
remaining two are competed on hard courts.
● Novak Djokovic and Margaret Court hold the record for most grand slam singles titles for men’s and
women’s tournaments respectively. So, statement 2 is not correct.

62
Vajiram & Ravi Prelims Test Series (2024)
Full Length Test – 07 – (R5537)
● A Golden Slam requires players to also win Olympic gold on top of the four grand slam events in the
same year. Steffi Graf was the first to complete a Golden Slam in 1988, with only wheelchair players
Diede de Groot and Dylan Alcott managing to replicate the feat since. So, statement 3 is not correct.
○ Wimbledon is the oldest grand slam event, founded in 1877, followed by the US Open four years
later, the French Open in 1891, and the Australian Open in 1905.
So, only one of the above statements is correct.
Therefore, option (a) is the correct answer.
Knowledge Box

● American Don Budge was the first to achieve a Grand Slam in 1938, with Australian
Rod Laver the only other men's singles player to repeat the feat.
● Maureen Connolly was the first woman to win a grand slam in women's singles when
she did so in 1953.
● Margaret Court and Steffi Graf are the only other players to have done the same since.
● Dylan Alcott was the last player to complete a Grand Slam in 2021 when he did so in
the wheelchair quad singles.
Relevance: Recently, Carlos Alcaraz defeated Novak Djokovic in the Wimbledon Finals of 2023.

63
Vajiram & Ravi Prelims Test Series (2024)
Full Length Test – 07 – (R5537)

You might also like